80

Click here to load reader

04 Poisoning

Embed Size (px)

DESCRIPTION

PREP poisoning collection of several years from American academy

Citation preview

Page 1: 04 Poisoning

American Academy of Pediatrics PREP 2014

American academy of pediatrics 1

Item 15 A 2-year-old boy is brought to the emergency department by ambulance. Thirty minutes

ago, his mother discovered him eating pills from a pillbox at his grandparents' house and

called 911. The boy's mother states that the pillbox contained a 1-week supply of his

grandparents' daily medications. She did not count how many tablets remained in the box

before the ambulance arrived, but she states, "I think only a couple were missing:' Both

grandparents take "blood pressure medicine" and that the grandfather takes "a pill for his

nerves"

The boy is well-appearing and playful. His vital signs are normal for his age, and you

note no abnormalities on physical examination. The mother states that she now feels

"silly for panicking over nothing. ' She asks you how soon she can take her son home.

Of the following, the BEST next step in managing this patient is

A. administration of activated charcoal at 1 g/kg

B. administration of intravenous normal saline at 20 mL/kg

C. discharge the boy after educating his mother about signs and symptoms to

observe for at home

D. observe the boy in the emergency department for development of symptoms

over the next 6 hours

E. perform gastric lavage to remove ingested pill fragments from the stomach

Dr_Faqehi

Page 2: 04 Poisoning

American Academy of Pediatrics PREP 2014

American academy of pediatrics 2

Item 15 S Preferred Response: A

The child described in this vignette was seen ingesting the contents of a pillbox thought

to contain an antihypertensive agent and an antidepressant, therefore, administration of

activated charcoal is warranted to decrease absorption of these toxins. Activated charcoal

minimizes absorption of drugs by binding them onto its surface; it has become the

gastrointestinal (GI) decontamination strategy of choice in pediatric patients and is most

effective when administered within the first hour after a toxic ingestion. The dose of

activated charcoal is 1 g/kg. Activated charcoal is contraindicated in patients with an

unprotected airway, patients with a disrupted GI tract, or patients in whom charcoal

therapy may increase the risk and severity of aspiration, such as in those ingesting a

hydrocarbon. Substances that are poorly adsorbed by activated charcoal include common

electrolytes, heavy metals such as iron, alcohols, cyanide, most solvents, and most water-

insoluble compounds. For the asymptomatic boy described in the vignette, no

contraindications for activated charcoal administration exist.

Poisoning represents one of the most common medical emergencies encountered by

young children and is responsible for a significant proportion of emergency department

visits in the adolescent population. More than 2 million toxic exposures are reported to

the American Association of Poi-son Control Centers Toxic Exposure Surveillance

System each year. Two-thirds of these exposures occur in individuals younger than 20

years of age, with half occurring in children younger than 6 years. All physicians caring

for children must be familiar with the evaluation and management of poisoning. While

poisonings in young children are usually unintentional, poisonings in adolescents and

young adults generally result from substance abuse, experimental risk-taking behaviors,

and depression or suicidal intent.

Numerous factors place young children at risk for unintentional poisonings. Between 1

and 2 years of age, most children learn to walk and develop the dexterity to use a pincer

grasp; a common way that children within this age group explore their environments is by

placing objects in their mouths. Young children like to mimic actions that they have seen

their family members perform, such as using household products and taking medications.

Furthermore, a number of household products and medications are brightly colored and

may even resemble candy, making them particularly attractive to children.

Most substances that young children are exposed to within their home environments,

such as cosmetics and personal care items, are nontoxic. Even among the significant

percentage of toxic exposures involving drugs, small quantities of most agents ingested

by a child require little treatment beyond reassurance. A few medications, however, can

be lethal to small children in quantities of only 1 or 2 pills or teaspoon-sized swallows.

Pediatric practitioners need to be familiar with the drug classes from which "one pill can

kill" when ingested by a toddler. These classes include cardio-vascular drugs (eg, -

blockers and calcium-channel antagonists), antidepressants, antipsychotics,

anticonvulsants, antiarrhythmic agents, salicylates, oral hypoglycemics, and opioids, all

of which are widely prescribed for adults. When drugs from these classes are involved,

Dr_Faqehi

Page 3: 04 Poisoning

American Academy of Pediatrics PREP 2014

American academy of pediatrics 3

proper evaluation and intervention are essential for preventing severe toxic effects and

even death in small children.

The first priority in managing any child who has ingested a toxic substance is to ensure

stability of the airway and take any necessary steps to maintain adequate ventilation and

circulation. The asymptomatic child who may have ingested only a few pills or swallows

of an unknown toxin presents a clinical dilemma. A careful history, physical

examination, and laboratory findings may narrow the differential diag-nosis and facilitate

an educated assessment of the potential severity of the exposure. In situations in which a

child may have ingested a medication with potentially lethal effects, the most appropriate

course of management is to decontaminate the child if no contraindications exist and to

monitor closely for a period of time, depending on the poison that may have been

ingested.

The boy described in the vignette is asymptomatic with normal vital signs; therefore,

administration of intravenous normal saline is not warranted. Since, he may have ingested

drugs with the potential to produce significant toxic effects within a few hours, GI

decontamination and a period of observation are required before discharge. Although

observation of the child for a period of several hours is warranted in this case,

administration of activated charcoal would be the best initial step in management because

activated char-coal is most efficacious within the first hour after ingestion. The clinical

benefit of gastric lavage has not been confirmed in controlled studies, and its routine use

in the management of poisoned patients is no longer recommended.

PREP Pearls

• Pediatric practitioners must recognize the drug classes from which "one pill

can kill" when ingested by a toddler.

• The first priority in managing a possible toxic ingestion is to ensure stability

of the airway and maintenance of adequate ventilation and circulation.

• Activated charcoal is the GI decontamination strategy of choice in pediatric

patients with possible toxic ingestions and should be given as soon as possible

provided there are no contraindications.

Dr_Faqehi

Page 4: 04 Poisoning

American Academy of Pediatrics PREP 2014

American academy of pediatrics 4

American Board of Pediatrics Content Specification(s):

• Understand the management of childhood poisonings

• Understand the management of poisonings by an unknown agent or by

multiple agents

Suggested Reading:

Braitberg G, Oakley E. Small dose ... big poison. Aust Fam Physician.

2010;39:826-833

Osterhoudt KC. The toxic toddler: drugs that can kill in small doses. Contemp

Pediatr. 2000;17:73

Osterhoudt KC, Ewald MB, Shannon M, Henretig FM. Toxicologic emergencies.

In: Fleisher GR, Ludwig S, eds. Textbook of Pediatric Emergency Medicine. 6th

ed. Philadelphia, PA: Lippincott Williams & Wilkins; 2010:1171-1223

Velez LI, Shepherd JG, Goto CS. Approach to the child with occult toxic

exposure. UpToDate. Available online only for subscription

Dr_Faqehi

Page 5: 04 Poisoning

American Academy of Pediatrics PREP 2014

American academy of pediatrics 5

Item 33 You are taking weekend telephone calls for your practice when the father of a 12-year-

old boy calls to let you know he is driving his son to the local hospital. The boy was

working in the family garage when he drank an unknown substance out of a plastic bottle

that he mistakenly thought contained water. Since swallowing some of the substance, the

boy has complained of severe throat pain, burning pain in his chest, difficulty

swallowing, and nausea. The father tells you that his son is drooling and that he can see a

few white "sores" on his tongue and the roof of his mouth.

Of the following, the MOST likely substance to cause the boy's symptoms and physical

findings is

A. antifreeze (ethylene glycol)

B. furniture polish (hydrocarbon)

C. insecticide (organophosphate)

D. toilet bowl cleaner (sodium hydroxide)

E. weed killer (glycophosphate)

Dr_Faqehi

Page 6: 04 Poisoning

American Academy of Pediatrics PREP 2014

American academy of pediatrics 6

Item 33 S Preferred Response: D

The adolescent boy described in the vignette is symptomatic after unintentional ingestion

of an unknown household product that was improperly stored in an unlabeled container.

Of the substances listed, sodium hydroxide, a corrosive product, is most likely

responsible for his symptoms.

Most unintentional ingestions in pediatric patients occur in their homes, where toxic

household products are often readily available. Many household cleaners, furniture

polishes, garden chemicals, and automotive products carry the risk of significant toxic

effects if ingested. Often, as in the vignette, a child's caregivers seek medical attention

when symptoms indicate toxic effects from exposure to an unknown product.

Recognition of the typical signs and symptoms related to specific toxic exposures is

critical to intervening appropriately and for providing appropriate anticipatory guidance.

Corrosives are concentrated acid, alkaline, or oxidizing agents. Many are ingredients

found within common household products, including toilet bowl cleaners, laundry

detergents, stain and mildew removers, floor cleaners, oven cleaners, rust removers,

phenol-based disinfectants, swimming pool products, and batteries. The Centers for

Disease Control and Prevention estimate that more than a half million children are treated

emergently each year for acute poisoning with corrosives. Ingestion of even small

amounts of a corrosive alkaline substance, such as toilet bowl cleaner, drain cleaners, or

rust remover, can result in serious penetrating injuries to mucosal and skin surfaces by

liquefaction necrosis. Acidic substances tend to cause injury via coagulation necrosis,

rather than liquefaction necrosis; thus, acids carry a lower risk of esophageal perforation

but are still capable of producing serious corrosive injury.

Patients ingesting corrosive substances typically present with odynophagia, dysphagia,

drooling, intraoral burns, or ulcerations, as experienced by the adolescent in the vignette.

Additional findings may include vomiting with hematemesis, respiratory difficulty with

stridor or wheezing, hoarseness, retrosternal chest pain, dyspnea, and burns on the face,

hands, or chest. Significant burns to the eyes may occur with any ocular exposure.

Because the primary mode of injury from contact with these toxins is direct tissue

corrosion, systemic symptoms are rare, and hemodynamic instability generally does not

occur. Apart from distress due to pain, mental status is usually normal in affected

children. The most pressing clinical concern after a caustic ingestion is the potential for

airway or esophageal injury. Early airway visualization and protection are indicated in

any patient presenting with stridor or respiratory distress. Evaluation of the esophagus by

upper endoscopy is indicated in patients who have intraoral burns, other symptoms

related to the ingestion, or a history strongly suggesting ingestion of a corrosive product.

In asymptomatic patients in whom caustic ingestion is uncertain, the need for endoscopy

is controversial.

Ethylene glycol, the toxic ingredient in antifreeze, presents with a clinical syndrome

typical of all alcohols. Depressed mental status is the initial manifestation, with nausea

and vomiting as common associated symptoms. Because of accumulation of the toxic

metabolites glycolaldehyde, glycolic acid, and oxalic acid, produced by the metabolism

Dr_Faqehi

Page 7: 04 Poisoning

American Academy of Pediatrics PREP 2014

American academy of pediatrics 7

of ethyl-ene glycol, severe metabolic acidosis ensues after ingestion. Seizure and coma

may manifest within a few hours after a significant ingestion. Affected patients may

progress to frank coma and cardiopulmonary failure. Hypocalcemia is another common

finding, resulting from the formation of calcium oxalate crystals by toxic metabolites;

these calcium oxylate crystals may be deposited in all organs of the body. Crystalluria in

patients with ethylene glycol poisoning is a late finding. Approximately 24 to 72 hours

after ingestion, patients typically progress to renal failure, and hemodialysis may be

warranted.

Glycophosphate-containing weed killers are irritants that may cause chemical

conjunctivitis, cough with inhalation, and vomiting after ingestion. These products are

not corrosive and generally do not cause mucosal injury.

Toxicity from insect repellants that contain organophosphate is marked by clinical signs

and symptoms related to the overactivation of cholinergic receptors by excess acetyl-

choline. The classic features of cholinergic toxicity can be recalled using the mnemonic

SLUDGE: salivation, lacrimation, urination, defecation or diarrhea, gastrointestinal

upset, and emesis. Among the expected vital sign abnormalities are tachypnea and

bradycardia. Pupillary constriction is a classic finding with organophosphate toxicity.

Ingestion of hydrocarbon -based products, such as furniture polish, primarily causes

respiratory distress due to pulmonary aspiration. Patients present with acute coughing,

gagging, and choking. Physical examination findings may include fever, tachypnea,

cyanosis, and abnormal lung sounds, which may include crackles and wheezing. Direct

central nervous system effects of the hydrocarbon may lead to lethargy, seizure, or even

coma.

PREP Pearls

Ingestion of even small amounts of a corrosive alkaline substance can result in

serious penetrating mucosal and skin injuries.

Patients ingesting corrosive substances typically present with odynophagia,

dysphagia, drooling, intraoral burns, or ulcerations.

The most pressing clinical concern after a caustic ingestion is the potential for

airway or esophageal injury.

All household chemical products should be stored in their original labeled

containers and kept out of the reach of children.

American Board of Pediatrics Content Specification(s):

Know the common household sources of acids and alkali

Suggested Reading:

Brunnie C, Savage RR. Corrosive ingestions. Pediatr Rev. 2006;27:154-155.

doi:10.1542/pir.27-4-154

Ferry GD, Fishman DS. Caustic esophageal injury in children. UptoDate.

Available online only for subscription

33-3 Osterhoudt, KC, Ewald MB, Shannon M, Henretig FM. Toxicologic

emergencies. In: Fleisher GR, Ludwig S, eds. Textbook of Pediatric Emergency

Medicine. 6th ed. Philadelphia, PA: Lippincott Williams & Wilkins; 2010:1171-

1223

Dr_Faqehi

Page 8: 04 Poisoning

American Academy of Pediatrics PREP 2014

American academy of pediatrics 8

Item 77 A 3-year-old boy is brought to the emergency department (ED) 30 minutes after ingesting

one of his grandmother's medications. The boy's mother reports that the grandmother

takes medications for depression and hypertension. During the first hour of being

monitored in the ED, the boy becomes increasingly lethargic and develops mydriasis and

a dry mouth. You notice on the cardiopulmonary monitor that his heart rate is 130

beats/min and he now appears to have a prolonged QRS duration. An electrocardiogram

demonstrates a QRS duration of 110 milliseconds, right axis deviation, and prolonged PR

and QT intervals.

Of the following, the MOST likely medication ingested by the patient is

A. amitriptyline

B. clonidine

C. fluoxetine

D. labetalol

E. nicardipine

Dr_Faqehi

Page 9: 04 Poisoning

American Academy of Pediatrics PREP 2014

American academy of pediatrics 9

Item 77 S Preferred Response: A

Tricyclic antidepressants (TCAs), such as amitriptyline and imipramine are widely used

for treating depression, chronic pain, and attention-deficit disorders. Pediatric patients

can be exposed to potential accidental or intentional ingestions of their own, their

siblings, or an adult relative's medications. Tricyclic antidepressants have strong

anticholinergic activity and can produce symptoms such as dry mouth, blurred vision,

tachycardia, urinary retention, constipation, dizziness, and vomiting. Overdoses of TCAs

can produce other more serious side effects, including central nervous system

abnormalities (irritability, lethargy, coma, and seizures), respiratory depression,

hypotension, and cardiac dysrhythmias. Symptoms may occur as early as 30 minutes and

are usually seen within 6 hours of ingestion.

Electrocardiographic changes (secondary to sodium channel blockade) are usually seen

within 6 hours of ingestion and include widened QRS interval, right axis deviation,

prolonged PR interval (first-degree heart block), abnormal T waves and ST segments, or

atrioventricular block. These changes may persist for days. Sinus tachycardia is the most

common rhythm but more serious cardiac complications such as bradydysrhythmias,

supraventricular tachycardia, ventricular tachycardia, and ventricular fibrillation may

occur within 24 hours of ingestion. A QRS duration greater than 100 milliseconds and

right axis deviation appear to be the strongest predictors of cardiac toxicity.

Clonidine overdoses are also of significant pediatric toxicology concern. Central nervous

system depression, pinpoint pupils, hypotension, bradydysrhythmias, and respiratory

depression are characteristic findings. The electrocardiographic changes seen in the

patient in the vignette are not consistent with a clonidine overdose. Ingestion of selective

serotonin reuptake inhibitors (SSRIs), such as fluoxetine, is associated with less toxicity

compared to TCAs. Agitation, delirium, tachycardia, hyperthermia, and muscle rigidity

may be seen, but again the electrocardiographic changes exhibited by the patient in the

vignette are not consistent with an SSRI overdose. β-blocker overdoses, such as that

caused by labetalol, can produce both central nervous system changes (lethargy, coma,

and seizures) as well as electrocardiographic changes, including a widening of the QRS

interval and prolongation of the PR interval. Dysrhythmias seen can include ventricular

tachycardia and asystole. However, the characteristic anticholinergic symptoms and signs

of TCA overdose would not be present. Overdoses of calcium channel blockers, such as

nicardipine, cause myocardial depression and cardiac conduction changes, including PR

interval prolongation and bradydysrhythmias. The anticholinergic and central nervous

symptoms that are seen with TCA overdose are not seen in calcium channel poisoning.

Dr_Faqehi

Page 10: 04 Poisoning

American Academy of Pediatrics PREP 2014

American academy of pediatrics 10

PREP Pearls

• Pediatric patients can be exposed to accidental ingestions of their own, their

siblings', or an adult relative's medications.

• Overdoses of tricyclic antidepressants (TCAs) can cause central nervous

system abnormalities, respiratory depression, hypotension, and cardiac

dysrhythmias.

• An electrocardiogram demonstrating a QRS duration greater than 100

milliseconds and right axis deviation appears to be the strongest predictor of

cardiac toxicity in TCA overdoses.

American Board of Pediatrics Content Specification(s):

• Understand that a danger of tricyclic antidepressant treatment is accidental

ingestion by siblings

• Understand that cardiac dysrhythmias may occur late after ingestion of

tricyclic antidepressants

Suggested Reading:

O'Donnell KA, Ewald MB. Poisonings. In: Kliegman RM, Stanton BMD, St

Geme J, Schor N, Behrman RE, eds. Nelson Textbook of Pediatrics. 19th ed.

Philadelphia, PA: Elsevier Saunders; 2011: 264

Traub SJ. Tricyclic antidepressant poisoning. UptoDate. Available online only for

subscription

Dr_Faqehi

Page 11: 04 Poisoning

American Academy of Pediatrics PREP 2014

American academy of pediatrics 11

Item 165 A 3-year-old girl is seen in the emergency department 30 minutes after ingesting a large

amount of her father's propranolol that was prescribed for the treatment of hypertension.

You are working with a group of medical students who ask you what symptoms the

patient may exhibit.

Of the following, the MOST likely symptom that would be seen is

A. hyperglycemia

B. hypertension

C. seizures

D. tachycardia

E. tachypnea

Dr_Faqehi

Page 12: 04 Poisoning

American Academy of Pediatrics PREP 2014

American academy of pediatrics 12

Item 165 Preferred Response: C

Beta-blocking agents are used to treat various adult and pediatric medical conditions,

including hypertension, heart failure, ischemic heart disease, dysrhythmias, migraine

headaches, and thyrotoxicosis. Cardiac side effects that can be seen at therapeutic doses

are hypotension, sinus bradycardia, or heart block. Noncardiac effects can include

increased airway resistance, hypoglycemia, and hyperkalemia. Especially in pediatric

patients, these findings may be accompanied by respiratory depression and changes in

mental status, including coma, delirium, and seizures.

Symptoms vary depending on the β-blocker ingested; propranolol is particularly likely to

cause central nervous system symptoms including seizures. The onset of symptoms

generally is within 2 hours of ingestion and almost always within .6 hours unless the drug

is a sustained-release formulation. As a result, patients with a known or suspected

ingestion of an overdose should be observed for 6 hours (24 hours for a sustained-release

formulation). Symptomatic patients should be admitted to an intensive care unit for

monitoring and treatment.

PREP Pearls

• β -blocker overdose can cause hypotension, sinus bradycardia, heart block,

increased airway resistance, respiratory depression, hypoglycemia, and

hyperkalemia.

• β -blocker overdose can produce changes in mental status, including coma,

delirium, and seizures.

• Symptoms of (β -blocker overdose are almost always seen by 6 hours unless

the drug is a sustained-release formulation.

American Board of Pediatrics Content Specification(s):

• Recognize common side effects of beta-blocking drugs

Suggested Reading:

• Lemkin E, Barrueto F. Beta blocker poisoning. UptoDate. Available online

only for subscription

• O'Donnell KA, Ewald MB. Poisonings. In: Kliegman RM, Stanton BMD, St

Geme J, Schor N, Behrman RE, eds. Nelson Textbook of Pediatrics. 19th ed.

Philadelphia, PA: Elsevier Saunders; 2011: 261

• Podrid PJ. Major side effects of beta blockers. UptoDate. Available online

only for subscription

Dr_Faqehi

Page 13: 04 Poisoning

American Academy of Pediatrics 2013 PREP SA on CD-ROM

Copyright 2013 © American Academy of Pediatrics 1

Question: 40

The mother of a 2-year-old boy calls you because she just found her son in the bathroom holding an empty bottle of liquid acetaminophen. The bottle had been full and she suspects he drank all of it, although there is some spillage on his shirt. She reports that she put him in bed for a nap about 90 minutes ago and does not know when he may have consumed the acetaminophen. He has had no vomiting and is behaving normally. Of the following, the MOST appropriate next step is toA. provide reassurance that no further evaluation is necessary

B. recommend that she administer ipecac at home

C. recommend that she call 911 for emergent transport to the emergency department

D. refer her to the emergency department for serum acetaminophen level testing

E. refer her to the emergency department for urine toxicology testing

Dr_Faqehi

Page 14: 04 Poisoning

American Academy of Pediatrics 2013 PREP SA on CD-ROM

Copyright 2013 © American Academy of Pediatrics 2

Preferred Response: DCritique: 40

The child in the vignette has likely ingested an unknown quantity of acetaminophen. While toddlers who ingest acetaminophen rarely consume harmful quantities, the risk of serious sequelae from toxic overdoses is significant. For this reason and because clinical signs and symptoms of a dangerous overdose are often absent, an acetaminophen level should be obtained before reassurance can be given that there will be no adverse effects. Urine toxicologic testing is not helpful in a known, single-agent, acetaminophen ingestion because the drug is not excreted in the urine. Ipecac syrup is no longer recommended for the treatment of toxic ingestions because of lack of efficacy. Unless the local emergency medical services can administer activated charcoal in the field, there is no likely benefit from emergency transport to the emergency department in this otherwise asymptomatic patient. Acetaminophen is the most commonly used antipyretic and analgesic in the United States and, as a result of its ready availability in most households, is responsible for more overdoses and overdose deaths than any other medication. The most important adverse effect is acute liver failure, which leads to approximately 250 deaths per year. Timely recognition of a toxic ingestion and administration of the antidote, N-acetylcysteine (NAC), can prevent morbidity and mortality. The clinical course after an acetaminophen overdose has 4 phases. In the first 24 hours after ingestion, symptoms, if there are any, are frequently vague and nonspecific. Nausea, vomiting, and malaise are most often reported. Twenty-four to 72 hours later, evidence of hepatotoxicity becomes apparent with abnormalities in liver synthetic function and transaminase increases. On examination, patients may have hepatomegaly and right upper quadrant tenderness. Liver failure, renal failure, and multiorgan failure develop and worsen between 72 and 96 hours, with most of the deaths occurring during this phase. Liver transplantation in this phase can prevent mortality. The final phase is the recovery period, during which patients who have not succumbed will clinically improve, although laboratory abnormalities and liver histology may not normalize for several months. Certain patients are at increased risk of serious adverse effects after acetaminophen overdose, including patients over 5 years of age, patients who have ingested repeated excessive doses, patients with Gilbert syndrome, and patients taking drugs that induce cytochrome P450 2E1 (CYP2E1) enzymes, such as anticonvulsants and antituberculous drugs. Evaluation and management of these ingestions begins with quantifying the ingested dose if possible. In patients without other risk factors, single acute doses of less than 150 mg/kg are generally not associated with toxicity. If the amount is unknown or greater than 150 mg/kg and if fewer than 4 hours have elapsed since the ingestion, the patient should be given activated charcoal as a decontaminant. A serum level should then be obtained between 4 and 24 hours and the patients level should be plotted on the Rumack-Matthew nomogram to determine the likelihood of hepatotoxicity. If the serum level suggests possible or probable hepatotoxicity, the patient should be treated with NAC, a glutathione precursor. Because the liver injury is caused by acetaminophen metabolites produced in the absence of glutathione, increasing the availability of glutathione decreases the production of the toxic metabolites and limits liver injury. Ideally, NAC should be administered within 10 hours of ingestion and should be started before levels are available in unknown or suspected large ingestions. The drug can be

Dr_Faqehi

Page 15: 04 Poisoning

American Academy of Pediatrics 2013 PREP SA on CD-ROM

Copyright 2013 © American Academy of Pediatrics 3

intravenously and discontinued if the level suggests no or low risk of hepatotoxicity. It is not possible to use the nomogram to predict hepatotoxicity in chronic overdoses or ingestions of sustained-release products. Consultation with the local poison control center is advised in these situations. SUGGESTED READING: Dyer KS. Acetaminophen (paracetamol) poisoning in children and adolescents. UptoDate Online. http://www.uptodate.com/contents/acetaminophen-paracetamol-poisoning-in-children-and-adolescents?source=search_result&search=acetaminophen+poisoning&selectedTitle=3%7E41 Wolf SJ, Heard K, Sloan EP, Jagoda AS; American College of Emergency Physicians. Clinical policy: critical issues in the management of patients presenting to the emergency department with acetaminophen overdose. Ann Emerg Med. 2007;50(3):292-313. doi:10.1016/j.annemergmed.2007.06.014

Dr_Faqehi

Page 16: 04 Poisoning

American Academy of Pediatrics 2013 PREP SA on CD-ROM

Copyright 2013 © American Academy of Pediatrics 4

Question: 125

A 2 year-old boy is brought to the emergency department after he was found in the bathroom with multiple open bottles scattered around him. His mother reports that there were pills and liquids spilled all over the floor. She is unable to provide a list of the medications. On physical examination, he is poorly responsive and moans to painful stimuli. He has a heart rate of 140 beats/min, a respiratory rate of 36 breaths/min, blood pressure of 90/60 mm Hg, and a temperature of 38.1°C. His pupils are midsized and sluggishly reactive. While you are examining him, he has a generalized tonic-clonic seizure. Of the following, the over-the-counter product MOST likely responsible for this childs symptoms isA. antidiarrheal medication containing bismuth subsalicylate

B. cold medication containing dextromethorphan

C. fever reducer containing acetaminophen

D. mouthwash

E. nasal spray containing oxymetazoline

Dr_Faqehi

Page 17: 04 Poisoning

American Academy of Pediatrics 2013 PREP SA on CD-ROM

Copyright 2013 © American Academy of Pediatrics 5

Preferred Response: ACritique: 125

The over-the-counter (OTC) product most likely to have produced this childs symptoms is an antidiarrheal medication containing bismuth subsalicylate. Acute salicylism is characterized by several signs and symptoms exhibited by the patient, including altered mental status ranging from lethargy to coma, seizures, tachypnea or hyperpnea, hyperpyrexia, and tachycardia. Over-the-counter cold medications typically contain stimulants like phenylephrine and lead to agitation, tachycardia, and hypertension. If the product contains dextromethorphan, the boys' pupils would be miotic secondary to opioid effects produced by a toxic dose of this medication. Mouthwashes typically contain ethanol, which would cause sedative-hypnotic adverse effects, such as lethargy, bradycardia, bradypnea, and hypotension. Nasal and ocular drops containing oxymetazoline can lead to coma and seizures, respiratory depression, and hypotension. Acute overdoses of acetaminophen may cause nausea and vomiting, but most patients remain asymptomatic. In 2010, the American Association of Poison Control Centers (AAPCC) reported that almost 35% of calls related to exposures in children less than 5 years old were due to OTC products. These included cosmetics and personal care items (13%), analgesics (9%), topical preparations (7%), and cough or cold preparations (5%). While most of these exposures were not serious, some did result in morbidity and mortality. Parents often assume that medications obtained without a prescription are nontoxic and are unaware that OTC products and medications may contain substances that can be harmful (Item C125), especially to small children. Anticipatory guidance around poisoning prevention should include safe storage recommendations for OTC products, many of which are commonly found in most households. SUGGESTED READING: Barnett AK, Boyer EW. Salicylate poisoning in children and adolescents UptoDate Online. http://www.uptodate.com/contents/salicylate-poisoning-in-children-and-adolescents?source=search_result&search=salicylate+toxicity&selectedTitle=2%7E57 Bronstein AC, Spyker DA, Cantilena LR, Green JL, Rumack BH, Dart RC. 2010 Annual Report of the American Association of Poison Control Centers' National Poison Data System (NPDS): 28th annual report. Clin Toxicol (Phila). 2011;49(10):910-941. doi:10.3109/15563650.2011.635149 Legano L. Alcohol. Pediatr Rev. 2007;28(4):153-155. doi:10.1542/pir.28-4-153 Rosenbaum C, Boyer EW. Dextromethorphan poisoning: epidemiology, pharmacology and clinical features. UptoDate Online. http://www.uptodate.com/contents/dextromethorphan-poisoning-epidemiology-pharmacology-and-clinical-features?source=search_result&search=over-the-counter+medication+toxicity&selectedTitle=10%7E150 Schillie SF, Shehab N, Thomas KE, Budnitz DS. Medication overdoses leading to emergency

Dr_Faqehi

Page 18: 04 Poisoning

American Academy of Pediatrics 2013 PREP SA on CD-ROM

Copyright 2013 © American Academy of Pediatrics 6

department visits among children. Am J Prev Med. 2009;37(3):181-187. doi:10.1016/j.amepre.2009.05.018

Dr_Faqehi

Page 19: 04 Poisoning

Dr_Faqehi

Page 20: 04 Poisoning

American Academy of Pediatrics 2013 PREP SA on CD-ROM

Copyright 2013 © American Academy of Pediatrics 8

Question: 166

You are called to the emergency department to evaluate a 3-year-old girl who was rescued by paramedics from a house fire. She was found unconscious at the scene. On arrival at the emergency department, she is difficult to arouse but is in no respiratory distress, and her breath sounds are clear bilaterally. She has no obvious external burns although there is soot around her nares. She has a temperature of 37.0°C, a heart rate of 130 beats/min, a respiratory rate of 24 breaths/min, and her blood pressure is 90/60 mm Hg. Her oxygen saturation by pulse oximetry is 97% on 100% oxygen administered by non-rebreather mask. Of the following, the MOST likely cause of her depressed neurologic condition isA. acute respiratory distress syndrome

B. airway edema

C. carbon monoxide exposure

D. methemoglobinemia

E. shock

Dr_Faqehi

Page 21: 04 Poisoning

American Academy of Pediatrics 2013 PREP SA on CD-ROM

Copyright 2013 © American Academy of Pediatrics 9

Preferred Response: CCritique: 166

The child described in the vignette has been rescued from a house fire and therefore is at high risk of carbon monoxide poisoning, as evidenced by her altered level of consciousness and soot around the nares. Although she is tachycardic, she shows no signs of respiratory distress or impaired perfusion; therefore, acute respiratory distress syndrome, airway edema, and shock are unlikely causes of her depressed neurologic condition. Methemoglobinemia after fire exposure is rare but has been reported. Patients generally present with intense cyanosis and respiratory distress in addition to a depressed neurologic state. Carbon monoxide is the leading cause of death by accidental poisoning in the United States, with over 500 fatalities each year. Carbon monoxide has extremely high affinity for hemoglobin, and exposure can result in impaired oxygen transport and release and subsequent tissue hypoxia. Initial symptoms after exposure include nausea, fatigue, confusion, and headache. The resultant hypoxia from the impaired oxygen transport and release can lead to coma, seizures, myocardial depression with hypotension, and dysrhythmia. Because pulse oximetry is unable to distinguish oxyhemoglobin and carboxyhemoglobin, it is unreliable in testing for carbon monoxide poisoning. If carbon monoxide exposure is suspected, direct measurement of a blood carboxyhemoglobin concentration should be performed using co-oximetry. Chest radiography and electrocardiography also should be performed. Although symptoms increase in severity as systemic carboxyhemoglobin levels increase, therapy should be guided by symptoms, not levels. Administration of 100% oxygen via a non-rebreathing mask should be administered while confirming the diagnosis because 100% oxygen shortens the half-life of carboxyhemoglobin to 40 to 80 minutes, as compared with 4 to 6 hours without therapy. Hyperbaric oxygen therapy further shortens the half-life to 15 to 30 minutes. Indications for its use include altered mental status, cardiac ischemia or dysrhythmias, carboxyhemoglobin levels greater than 25% (or >15% if pregnant), or persistent symptoms after 4 to 6 hours after treatment with normobaric oxygen. Endotracheal intubation should be performed in patients who are unable to protect their airway because of significantly depressed neurologic symptoms or in patients who have evidence of significant thermal injury of the respiratory tract. SUGGESTED READING: ODonnell KA, Ewald MB. Poisonings: toxic gases. In: Kliegman RM, Behrman RE, Jenson HB, Stanton BF, eds. Nelson Textbook of Pediatrics. 19th ed. Philadelphia, PA: Saunders Elsevier; 2011:269-270 Zimmerman JL. Poisonings and overdoses in the intensive care unit: general and specific management issues. Crit Care Med. 2003;31(12):2794-2801

Dr_Faqehi

Page 22: 04 Poisoning

American Academy of Pediatrics 2013 PREP SA on CD-ROM

Copyright 2013 © American Academy of Pediatrics 10

Question: 250

You are working in the emergency department and evaluating a 2-year-old boy who was brought by his parents because of ingestion several hours ago of an unknown liquid that was being stored in the garage. The parents report the boy has had several episodes of emesis since the ingestion. The childs temperature is 37.0°C, heart rate is 100 beats/min, respiratory rate is 30 breaths/min, and blood pressure is 78/45 mm Hg. His oxygen saturation by pulse oximetry is 95% on oxygen administered at 2 L/min via nasal cannulae. Physical examination reveals the boy to be sleepy and diaphoretic; his pupils are 2 mm and mildly reactive and he is drooling. His abdomen is tender to palpation and there are hyperactive bowel sounds. Of the following, the toxin MOST likely to produce this boys symptoms isA. bleach

B. ethylene glycol

C. kerosene

D. methanol

E. organophosphates

Dr_Faqehi

Page 23: 04 Poisoning

American Academy of Pediatrics 2013 PREP SA on CD-ROM

Copyright 2013 © American Academy of Pediatrics 11

Preferred Response: ECritique: 250

Organophosphates and carbamates are commonly used insecticides. They present a significant pediatric toxicology problem because of accidental ingestion of improperly stored products. Worldwide, there are an estimated 3 million cases yearly with 300,000 deaths, while in the United States there are 8,000 cases with fewer than 15 deaths per year. In addition, these compounds are nerve agents that have been used in warfare and terrorist activities, such as the 1995 Tokyo subway attacks. Organophosphates and carbamates prevent the breakdown of acetylcholine through the inhibition of cholinesterases, including red blood cell acetylcholinesterase, plasma cholinesterase (pseudocholinesterase), and neuropathy target esterase. Unlike organophosphates that form permanent bonds with cholinesterases, carbamates form temporary bonds and, as a result, produce symptoms that are less severe and of shorter duration. In addition, carbamates do not penetrate the central nervous system. Symptoms caused by these agents result from the accumulation of acetylcholine at both the nicotinic and the muscarinic receptors. Muscarinic symptoms include tearing, drooling, diaphoresis, emesis, urinary and fecal incontinence, bronchospasm, miosis, bradycardia, and hypotension. Nicotinic symptoms include weakness, fasciculations, hypertension, and tachycardia. Central nervous symptoms include confusion, delirium, seizures, and coma. Two common mnemonics (Item C250) are used to recall the muscarinic symptoms, but it is important to remember that children often present with central nervous system and nicotinic effects and therefore pose a diagnostic challenge. Results from laboratory studies are normal. Levels of serum or red blood cell cholinesterase activity can confirm the diagnosis but are often not available in a timely fashion and, as a result, appropriate clinical suspicion is critical for diagnosis and initiation of treatment. If readily available, serial measurements of red blood cell cholinesterase activity (but not serum cholinesterase) can be used to assess the severity of exposure and guide therapy. Treatment of organophosphate and carbamate poisonings consists of decontamination, supportive care, and antidote treatment advised by a toxicologist. Decontamination consists of washing the skin while avoiding hypothermia. Precautions should be made to protect healthcare personnel from exposure during the decontamination. Activated charcoal administered within 1 hour of ingestion has been recommended, but gastric lavage is generally avoided because of the risk of aspiration in the setting of concurrent respiratory distress and altered neurologic status. Supportive care may include intravenous fluids, intubation (succinylcholine, a depolarizing neuromuscular blocker, is contraindicated) and mechanical ventilation, vasopressors, and seizure control (benzodiazepines are the treatment of choice). Specific treatment consists of atropine to counteract the actions of the excess acetylcholine and an oxime (eg, pralidoxime [2-PAM]) to reactivate cholinesterases. Repeat doses of atropine are indicated as long as muscarinic symptoms persist. 2-PAM is most effective if given within 24 to 48 hours because of the “aging” of the acetylcholinesterase-organophosphate binding that becomes irreversible with time. 2-PAM is generally not indicated in carbamate poisonings because the compound-enzyme bond will degrade spontaneously. Atropine and 2-PAM are usually given as a loading dose and followed by continuous infusion. Both medications are critical components of the United States Strategic National Stockpile and similar storage programs in other countries. They exist in the form of auto-injectors for potential terrorist attacks; however, injectors with proper pediatric dosing do not exist. Recovery can be

Dr_Faqehi

Page 24: 04 Poisoning

American Academy of Pediatrics 2013 PREP SA on CD-ROM

Copyright 2013 © American Academy of Pediatrics 12

organophosphate poisoning with persistent neurotoxicity, muscular weakness, polyneuropathies, and respiratory failure. Organophosphate poisonings may be difficult to diagnose because the symptoms mimic those of mushroom, nicotine, and opioid poisonings. Methanol and ethylene glycol can cause cardiorespiratory and central nervous system symptoms, including hypotension, respiratory failure, coma, seizures, nausea, and vomiting. However, lacrimation and miosis (mydriasis is the more common finding), as exhibited by the boy in the vignette, are atypical. In both ingestions, laboratory studies will show a marked metabolic acidosis with a serum osmolar gap. Kerosene ingestion can produce respiratory and abdominal symptoms but miosis is not present. In addition, gagging is typically observed after hydrocarbon ingestion. Ingestion of bleach or other caustic chemicals produces oral or esophageal burns associated with gagging, drooling, vomiting, and refusal to swallow. These symptoms were not exhibited by the boy in the vignette. SUGGESTED READING: Bird S. Organphosphate and carbamate poisoning. UpToDate Online. http://www.uptodate.com/contents/organophosphate-and-carbamate-poisoning?source=search_result&search=organophosphates&selectedTitle=1%7E16 Fox J, Woods SK, Gundacaram A, Varma SK, Huang YH, Wong KS. Index of suspicion. Pediatr Rev. 2005:26(7):263-270. doi:10.1542/pir.26-7-263 ODonnell KA, Ewald MB. Poisonings: cholinesterase-inhibiting insecticides. In: Kliegman RM, Stanton BF, St Geme JW III, Schor NF, Behrman RE, eds. Nelson Textbook of Pediatrics. 19th ed. Philadelphia, PA: Saunders Elsevier; 2011:266-267

Dr_Faqehi

Page 25: 04 Poisoning

Dr_Faqehi

Page 26: 04 Poisoning

American Academy of Pediatrics 2012 PREP SA on CD-ROM

Copyright 2012 © American Academy of Pediatrics 1

Question: 12

You are precepting a group of medical students and leading a discussion on vaccine-preventablediseases. One of the students asks you to describe how to differentiate varicella from smallpox in a childpresenting with a vesicular rash.

Of the following, the clinical feature MOST suggestive of smallpox is

A. abrupt onset of rash in a previously well child

B. centripetal spread of the skin lesions

C. involvement of the palms and soles with rash

D. lesions in multiple stages on the same part of the body

E. superficial nature of the skin vesicles

Dr_Faqehi

Page 27: 04 Poisoning

American Academy of Pediatrics 2012 PREP SA on CD-ROM

Copyright 2012 © American Academy of Pediatrics 2

Preferred Response: CCritique: 12

Although smallpox has been eradicated since 1977 through successful implementation of worldwidecontrol and vaccine programs, concern has been raised for the virus being a potential bioterrorismagent. Clinical features of smallpox and comparison to varicella are summarized (Item C12A). In contrast to varicella, the rash in smallpox involves the palms and soles. Smallpox infectionstypically are characterized by a 4- to 5-day prodrome of fever and malaise, and the infected individualappears very ill at the time of rash presentation. The lesions of smallpox spread centrifugally. Althoughlesions in varicella appear in multiple stages of development on the same part of the body (Item C12B),smallpox lesions are homogenous (Item C12C). Finally, smallpox lesions are deeper into the skincompared to the superficial "dew drop on a rose petal" appearance in varicella.

SUGGESTED READING:

American Academy of Pediatrics. Smallpox (variola). In: Pickering LK, Baker CJ, Kimberlin DW, LongSS, eds. Red Book: 2009 Report of the Committee on Infectious Diseases. 28th ed. Elk Grove Village,IL: American Academy of Pediatrics; 2009:596-598

Breman JG, Henderson DA. Diagnosis and management of smallpox. N Engl J Med. 2002;346:1300-1308. Accessed January 2011 at: http://www.nejm.org/doi/full/10.1056/NEJMra020025

Centers for Disease Control and Prevention. Emergency Preparedness & Response. Evaluating aRash Illness Suspicious for Smallpox. 2007. Accessed January 2011 at:http://www.bt.cdc.gov/agent/smallpox/diagnosis/riskalgorithm/

World Health Organization. Media Centre. Smallpox Fact Sheet. 2007. Accessed January 2011 at:http://www.who.int/mediacentre/factsheets/smallpox/en/index.html

Dr_Faqehi

Page 28: 04 Poisoning

Dr_Faqehi

Page 29: 04 Poisoning

Dr_Faqehi

Page 30: 04 Poisoning

Dr_Faqehi

Page 31: 04 Poisoning

American Academy of Pediatrics 2012 PREP SA on CD-ROM

Copyright 2012 © American Academy of Pediatrics 6

Question: 16

An 18-month-old boy is brought to the office because he has been difficult to arouse for the pasthour. His mother reports that earlier in the day she found him in his grandmother's room playing with hermedicine bottles, but none of the bottles were opened. The mother explains that the grandmother takes"pills for her heart." The child is somnolent and responsive only to pain. His temperature is 38.6°C, heartrate is 130 beats/min, respiratory rate is 56 breaths/min, and blood pressure is 90/60 mm Hg, and hispupils are midsized and reactive. The remainder of his physical examination findings are normal.

Of the following, the MOST likely explanation for this child's symptoms is

A. aspirin ingestion

B. intracranial hemorrhage

C. lisinopril ingestion

D. metoprolol ingestion

E. sepsis

Dr_Faqehi

Page 32: 04 Poisoning

American Academy of Pediatrics 2012 PREP SA on CD-ROM

Copyright 2012 © American Academy of Pediatrics 7

Preferred Response: ACritique: 16

The child described in the vignette is exhibiting signs and symptoms consistent with salicylatepoisoning. Although salicylate poisoning is not seen as commonly as before the introduction ofchildproof packaging and other nonsalicylate antipyretics/analgesics, intoxication with aspirin and othersalicylates is responsible for more than 20,000 reported cases to United States poison control centersannually. The recommended use of low-dose aspirin as primary prevention for cardiovascular disease inadults means that aspirin is readily available and potentially accessible to children in many households. The clinical findings of acute salicylism can be explained by the many cellular and systemic effects ofthe drugs (Item C16) and are related to ingested dose. Mild symptoms are typically seen in ingestions of300 mg/kg or less; those of greater than 300 mg/kg lead to moderate toxicity and greater than 500mg/kg lead to death. Early/mild signs and symptoms include fever/hyperpyrexia, nausea, vomiting,diarrhea, and tinnitus. Tachypnea/hyperpnea, tachycardia, hypoglycemia, metabolic acidosis with anelevated anion gap, altered mental status (lethargy to coma), and seizures are commonly seen in moresevere poisonings. Such clinical findings can overlap with a number of equally serious conditions, including diabeticketoacidosis, acute iron poisoning, and ethylene glycol toxicity. The patient who has intracranialhemorrhage is likely to exhibit bradycardia, hypertension, and focal neurologic signs. In lisinopril andmetoprolol ingestions, hypotension predominates, with bradycardia seen additionally with a beta-blockeroverdose. Finally, an obtunded patient who has sepsis is likely to exhibit hypotension and other findingsconsistent with septic shock or meningitis. The management phases of acute salicylate poisoning encompass stabilization, decontamination,and elimination, with all aspects of therapy directed at maintaining serum alkalemia. Because acidosispromotes diffusion of salicylic acid into the central nervous system, life-threatening neurologic symptomscan be avoided with aggressive alkalization. Initial stabilization should focus on airway maintenance,rehydration, and correction of electrolyte abnormalities. Oxygen should be provided as needed, butintubation and mechanical ventilation are reserved for those patients who have respiratory failure. Theincreases in minute ventilation seen in salicylate-poisoned patients contribute significantly tomaintenance of alkalemia, and attempts to normalize ventilation by eliminating the patient's ownhyperpnea have resulted in death from severe acidosis. Hypovolemia and potassium deficits should becorrected. Intravenous fluids should contain both potassium and sodium bicarbonate to repletepotassium stores and maintain alkalemia. Serum pH measurements should be maintained between 7.5and 7.6 during therapy. Decontamination is accomplished using multiple doses of activated charcoal, which should beadministered every 4 hours until the patient is asymptomatic and the salicylate concentration is less than30 mg/dL. Salicylic acid is eliminated in the urine and such elimination is maximized if the urine isalkaline. In severe poisoning associated with significant neurologic signs, pulmonary edema,unresponsive metabolic acidosis, renal failure, or plasma salicylate concentrations greater than 100mg/dL, hemodialysis is indicated.

SUGGESTED

Dr_Faqehi

Page 33: 04 Poisoning

American Academy of Pediatrics 2012 PREP SA on CD-ROM

Copyright 2012 © American Academy of Pediatrics 8

READING:

Barnett AK, Boyer EW, Traub SJ. Salicylate poisoning in children and adolescents. UpToDate Online18.3. 2010. Accessed January 2010 at:http://www.uptodate.com/online/content/topic.do?topicKey=ped_tox/8460

Chyka PA, Erdman AR, Christianson G, et al. Salicylate poisoning: an evidence-based consensusguideline for out-of-hospital management. Clin Toxicol (Phila). 2007;45:95-131. Accessed January 2011at: http://www.guideline.gov/content.aspx?id=9905

O'Donnell KA, Burns Ewald M. Pediatric drug therapy: poisonings. In: Kliegman RM, Stanton BF, St.Geme JW III, Schor NF, and Behrman RE, eds. Nelson Textbook of Pediatrics. 19th ed. Philadelphia,PA: Saunders Elsevier; 2011:250-270

Stolbach AI, Hoffman RS, Nelson LS. Mechanical ventilation was associated with acidemia in a caseseries of salicylate-poisoned patients. Acad Emerg Med 2008;15:866-869. DOI: 10.1111/j.1553-2712.2008.00205.x. Accessed January 2011 at: http://onlinelibrary.wiley.com/doi/10.1111/j.1553-2712.2008.00205.x/full

US Preventive Services Task Force. Aspirin for the prevention of cardiovascular disease: U.S.Preventive Services Task Force recommendation statement. Ann Intern Med. 2009;150:396-404.Accessed January 2011 at: http://www.annals.org/content/150/6/396.long

Dr_Faqehi

Page 34: 04 Poisoning

Dr_Faqehi

Page 35: 04 Poisoning

American Academy of Pediatrics 2012 PREP SA on CD-ROM

Copyright 2012 © American Academy of Pediatrics 10

Question: 34

A father brings his 2-year-old son to the emergency department after they had spent several hours inthe garage while the father worked on the car. The father reports that approximately 30 minutes ago heheard the child coughing and found him with an open bottle of charcoal lighter fluid in his hands. Onphysical examination, the awake and alert child's temperature is 37.0°C, heart rate is 120 beats/min,respiratory rate is 24 breaths/min, blood pressure is 90/60 mm Hg, and oxygen saturation is 98%. Hisshirt is saturated with lighter fluid. You remove the boy's shirt and decontaminate his skin.

Of the following, the MOST appropriate next step is to

A. obtain a STAT chest radiograph

B. obtain a urine toxicology screen

C. perform gastric lavage

D. place the child under observation

E. reassure the father and discharge the patient

Dr_Faqehi

Page 36: 04 Poisoning

American Academy of Pediatrics 2012 PREP SA on CD-ROM

Copyright 2012 © American Academy of Pediatrics 11

Preferred Response: DCritique: 34

The boy described in the vignette has ingested a hydrocarbon-containing substance. In contrast tomost other toxic ingestions, management does not typically involve decontamination or elimination butfocuses on respiratory stabilization and observation. The asymptomatic patient who has had ahydrocarbon exposure should be observed for the development of signs or symptoms for a minimum of6 hours. A chest radiograph should be obtained 4 to 6 hours after exposure. If the radiograph is normaland the patient does not develop any symptoms during the observation period, he or she may be safelydischarged. Hydrocarbons are a large family of compounds that most commonly cause toxic effects in thepulmonary and central nervous systems when ingested. Hydrocarbon ingestion does not often lead tosystemic toxicity unless other toxic substances such as camphor or pesticides are admixed with thehydrocarbon. Rather, aspiration during the swallowing event deposits the hydrocarbon in the pulmonarytree, where it directly injures the respiratory mucosa. The resulting damage leads to chemicalpneumonitis. Aspiration risk is highest with low-viscosity, high-volatility hydrocarbons such as kerosene,furniture polish, mineral spirits, and gasoline. Highly volatile hydrocarbons may diffuse rapidly into thecentral nervous system, leading to ataxia, somnolence, stupor, or coma. Clinical suspicion should be high for an aspiration event in any child who presents following ahydrocarbon exposure with a history of coughing/gagging or respiratory signs and symptoms such astachypnea, wheezing, or hypoxemia. Evaluation of symptomatic patients should include assessment ofoxygen saturation or arterial blood gasses and chest radiography. Urine toxicology screening is notroutinely helpful unless an illicit coingestant is suspected. A chest radiograph (Item C34) should beobtained acutely in a symptomatic patient, but because radiographic findings often lag behind clinicalfindings, the radiograph should be repeated in 4 to 6 hours if it appears initially normal. Supportive careshould be provided, including oxygen therapy, beta-agonist treatment for wheezing, andintubation/mechanical ventilation for respiratory failure. Antibiotics and corticosteroids are not indicated.Gastric lavage is contraindicated because it has the potential to cause further aspiration. Patients whohave any respiratory signs or symptoms on presentation should be observed for 24 to 48 hours fordisease progression.

SUGGESTED READING:

Levine MD, Greshem C III. Toxicity, hydrocarbons. eMedicine Specialties, Emergency Medicine,Toxicology. 2009. Accessed January 2011 at: emedicine.medscape.com/article/821143-overview

Lewander WJ, Aleguas A Jr. Hydrocarbon poisoning. UpToDate Online 18.1. 2010. AccessedJanuary 2011 at: http://www.uptodate.com/online/content/topic.do?topicKey=ped_tox/11453

O'Donnell KA, Burns Ewald M. Pediatric drug therapy: poisonings. In: Kleigman RM, Stanton BF, St.Geme JW III, Schor NF, and Behrman RE, eds. Nelson Textbook of Pediatrics. 19th ed. Philadelphia,PA:

Dr_Faqehi

Page 37: 04 Poisoning

American Academy of Pediatrics 2012 PREP SA on CD-ROM

Copyright 2012 © American Academy of Pediatrics 12

Saunders Elsevier; 2011:250-270

Dr_Faqehi

Page 38: 04 Poisoning

Dr_Faqehi

Page 39: 04 Poisoning

American Academy of Pediatrics 2012 PREP SA on CD-ROM

Copyright 2012 © American Academy of Pediatrics 14

Question: 86

An 18-month-old boy is brought to the emergency department after being found in his grandfather'sroom with several open pill bottles. The family reports that they removed two unidentifiable tablets fromhis mouth and found 23 more scattered on the floor. The medications include terazosin, simvastatin,aspirin, acetaminophen, and allopurinol. The sleepy but arousable child has a temperature of 37.0°C,heart rate of 160 beats/min, respiratory rate of 24 breaths/min, and blood pressure of 66/34 mm Hg. Theremainder of his physical examination findings are normal.

Of the following, the medication that is MOST likely to be the cause of this child's clinical findings is

A. acetaminophen

B. allopurinol

C. aspirin

D. simvastatin

E. terazosin

Dr_Faqehi

Page 40: 04 Poisoning

American Academy of Pediatrics 2012 PREP SA on CD-ROM

Copyright 2012 © American Academy of Pediatrics 15

Preferred Response: ECritique: 86

The boy described in the vignette is lethargic and hypotensive, presumably due to a toxic ingestion.Of the medications listed, only terazosin, an alpha-blocker used to treat symptoms associated withprostatic hypertrophy, causes hypotension. Many medications commonly used to treat various ofconditions in adults, from hypertension to insomnia, may be found in a child's environment and ingestedaccidentally. Hypotension following an ingestion can be an important clue to the identity of the ingestedagent (Item C86). When evaluating a child following an unknown ingestion, it is important to collect information that willhelp in identifying agents that may cause life-threatening symptoms or complications and have specifictreatments or antidotes. Assessment of vital signs, pupil size, skin, and neurologic status can provideuseful clues. More importantly, however, is initial stabilization of the patient and correction of any vitalsign derangements. The adage "treat the patient, not the poison" reminds the clinician that the treatmentof patients who have ingested toxic substances is primarily supportive, reactive, and often not specific tothe actual agent. A patient who is hypotensive following an unknown ingestion should be treated initially with fluidresuscitation. Fluid boluses of 20 mL/kg 0.9% saline should be administered rapidly, with carefulreassessment following each bolus. If the patient's blood pressure does not normalize after threeboluses, vasopressor agents such as dopamine or dobutamine should be considered.Electrocardiography and bedside glucose measurement also should be obtained to determine if thehypotension is related to a cardiac dysrhythmia or hypoglycemia.

SUGGESTED READING:

O'Donnell KA, Burns Ewald M. Pediatric drug therapy: poisonings. In: Kliegman RM, Stanton BF, St.Geme JW III, Schor NF, and Behrman RE, eds. Nelson Textbook of Pediatrics. 19th ed. Philadelphia,PA: Saunders Elsevier; 2011:250-270

Velez LI, Shepherd JG, Goto CS. Approach to the child with occult toxic exposure. UpToDate Online18.3. 2010. Accessed by subscription January 2011 at:http://www.uptodate.com/online/content/topic.do?topicKey=ped_tox/3023

Dr_Faqehi

Page 41: 04 Poisoning

Dr_Faqehi

Page 42: 04 Poisoning

American Academy of Pediatrics 2012 PREP SA on CD-ROM

Copyright 2012 © American Academy of Pediatrics 17

Question: 122

A 2-year-old boy is brought to the emergency department after his mother found him with an openbottle of toilet bowl cleaner. She reports that he had spilled some on his shirt and had some on his face,but she does not know if he drank any of it. The child is awake and alert, and his vital signs are normal.He is drooling slightly, but examination of his oropharynx reveals no lesions.

Of the following, the MOST appropriate next step is to

A. administer activated charcoal

B. administer syrup of ipecac

C. perform gastric lavage

D. provide no further treatment

E. refer the boy to a gastroenterologist for urgent endoscopy

Dr_Faqehi

Page 43: 04 Poisoning

American Academy of Pediatrics 2012 PREP SA on CD-ROM

Copyright 2012 © American Academy of Pediatrics 18

Preferred Response: ECritique: 122

Ingestion of a caustic substance causes injury to mucosal and skin surfaces by liquefaction necrosisin alkali exposures and by protein coagulation in acid exposures. Substances at the extremes of the pHscale (<2 and >12) are especially damaging. Patients typically present with drooling, dysphagia,odynophagia, and in many cases, intraoral burns. In addition, they may have vomiting withhematemesis; respiratory distress with stridor or wheezing; and burns on the face, hands, or chest.Because the primary mode of injury is direct tissue corrosion and systemic symptoms are rare,decontamination of patients following caustic ingestions is focused on washing the skin and flushing theeyes, if indicated. Use of activated charcoal is not indicated because it can make subsequentendoscopic evaluation of the esophagus difficult. Syrup of ipecac and gastric lavage are contraindicatedbecause of potential aspiration risk. Further, gastric lavage carries the risk of esophageal perforation. The major clinical concerns with a caustic ingestion are airway or esophageal injury. Severe gastritis,perforation, or late stricture formation also may result from significant ingestions. Early airwayvisualization and protection are indicated in any patient who presents with stridor or respiratory distress,and evaluation of the esophagus by upper endoscopy is indicated in patients who have intraoral burns orother symptoms. In addition, some asymptomatic patients, such as the boy in the vignette, should beconsidered endoscopy candidates, based on history or other clinical concerns. As many as 45% ofpatients who do not have oral burns and 12% of asymptomatic patients have findings on endoscopy.The need for endoscopy in the asymptomatic patient in whom a significant caustic ingestion isquestionable is controversial.

SUGGESTED READING:

Ferry GD. Caustic esophageal injury in children UpToDate Online 18.3. 2010 Accessed January2011 at: http://www.uptodate.com/online/content/topic.do?topicKey=pedigast/11441

Kardon EM. Toxicity, caustic ingestions. eMedicine Specialties, Emergency Medicine, Toxicity. 2010.Accessed January 2011 at: http://emedicine.medscape.com/article/813772-overview

O'Donnell KA, Burns Ewald M. Pediatric drug therapy: poisonings. In: Kleigman RM, Stanton BF, St.Geme JW III, Schor NF, and Behrman RE, eds. Nelson Textbook of Pediatrics. 19th ed. Philadelphia,PA: Saunders Elsevier; 2011:250-270

Dr_Faqehi

Page 44: 04 Poisoning

American Academy of Pediatrics 2012 PREP SA on CD-ROM

Copyright 2012 © American Academy of Pediatrics 19

Question: 162

A 5-year-old boy presents to the emergency department 30 minutes after he ingested some of hismother's tricyclic antidepressant. Over the ensuing hour of observation in the emergency department, hedevelops lethargy, irritability, and autonomic nervous system findings of mydriasis, dry mouth, andurinary retention. Within 3 hours of ingestion, these symptoms have resolved.

Of the following, the MOST appropriate next step in management is

A. chest radiography and arterial blood gas

B. discharge home without further evaluation

C. electrocardiography and continuous cardiac monitoring

D. serum electrolytes assessment

E. tricyclic serum drug concentration assessment and discharge home

Dr_Faqehi

Page 45: 04 Poisoning

American Academy of Pediatrics 2012 PREP SA on CD-ROM

Copyright 2012 © American Academy of Pediatrics 20

Preferred Response: CCritique: 162

An overdose of any of the tricyclic antidepressant drugs may result in a fatal cardiac dysrhythmia.Often, the effect on the heart is delayed; presentation to the emergency department with various signsand symptoms may precede the cardiac findings. Such earlier features may include lethargy, irritability,anticholinergic effects upon the autonomic nervous system associated with sympathetic nervous systemdysfunction, and seizures. Anticholinergic effects of tricyclic antidepressants include dry mouth, blurredvision, tachycardia, urinary retention, constipation, dizziness, and vomiting. The cardiac toxicity associated with this class of agents most often manifests as a prolongation of theQRS complex on electrocardiographic monitoring. Therefore, the boy described in the vignette shouldreceive electrocardiography and continuous cardiac monitoring. In addition, atrioventricular block canoccur, and prolongation of the PR interval (first-degree heart block) is common. The QRS complexprolongation is caused by delayed conduction through a poisoned myocardium. As toxic effects on themyocardium worsen, loss of cardiac mechanical activity and severely widened QRS complexes occursimultaneously. Ventricular tachyarrhythmias are a common late finding. Electromechanical dissociation(EMD) can result, leading to the need for urgent cardiopulmonary resuscitation. EMD does not respondto electrical cardioversion, instead requiring cardiac support with extracorporeal membrane oxygenationor some other form of ventricular assist device. If there is metabolic acidosis, infusion of sodiumbicarbonate is appropriate. Two mechanisms are postulated for its therapeutic effect. Tricyclicantidepressants are protein-bound and become less bound in more acidic conditions. By reversing theacidosis, protein binding increases and bioavailability decreases. An alternative explanation is that thesodium load helps to reverse the sodium channel-blocking effects of the tricyclic antidepressants.Treatment is otherwise supportive. Respiratory depression can result from the central nervous system sedative properties of theseagents, and if present, warrants careful laboratory and clinical observation. However, these findings donot appear late, and this boy has no indication of respiratory compromise. Discharging the boy after 3hours of observation in the emergency department is not appropriate, given the potential for late cardiaceffects. Although serum electrolyte imbalance could exacerbate any ventricular dysrhythmia, tricyclicantidepressant toxicity does not directly affect serum electrolyte concentrations or renal function. Thetricyclic antidepressants are highly metabolized by the cytochrome P450 hepatic enzymes. Measuringserum concentrations of the tricyclic antidepressants may be useful in assessing the potential fortoxicity, but such findings alone should not lend reassurance when early noncardiac symptoms arepresent. In this situation, further inpatient monitoring is always indicated.

SUGGESTED READING:

Kerr GW, McGuffie AC, Wilkie S. Tricyclic antidepressant overdose: a review. Emerg Med J.2001;18:236-241. DOI: 10.1136/emj.18.4.236. Accessed January 2011 at:http://www.ncbi.nlm.nih.gov/pmc/articles/PMC1725608/?tool=pubmed

Dr_Faqehi

Page 46: 04 Poisoning

American Academy of Pediatrics 2012 PREP SA on CD-ROM

Copyright 2012 © American Academy of Pediatrics 21

Rosenbaum T, Kou M. Are one or two dangerous? Tricyclic antidepressant exposure in toddlers. JEmerg Med. 2005;28:169-174. DOI: 10.1016/j.jemermed.2004.08.018. Abstract accessed January 2011at: http://www.ncbi.nlm.nih.gov/pubmed/15707813

Singh N, Singh KH, Khan IA. Serial electrocardiographic changes as a predictor of cardiovasculartoxicity in acute tricyclic antidepressant overdose. Am J Ther. 2002;9:75-79. Abstract accessed January2011 at: http://www.ncbi.nlm.nih.gov/pubmed/11782822

Dr_Faqehi

Page 47: 04 Poisoning

American Academy of Pediatrics 2012 PREP SA on CD-ROM

Copyright 2012 © American Academy of Pediatrics 22

Question: 179

A couple who is new to the community comes to you for a prenatal visit. They live in a home with aprivate well and have questions about the safety of providing well water to their newborn.

Of the following, you are MOST likely to advise them to use

A. boiled tap water

B. bottled distilled water

C. bottled drinking water

D. filtered tap water

E. tap water

Dr_Faqehi

Page 48: 04 Poisoning

American Academy of Pediatrics 2012 PREP SA on CD-ROM

Copyright 2012 © American Academy of Pediatrics 23

Preferred Response: ACritique: 179

In the United States, municipal water supplies, community wells, and private wells equipped withmodern purification systems are generally believed to safeguard the water supply for drinking. Up to onesixth of the homes in the United States have water supplied by private wells, as do some child carecenters and schools in suburban and rural areas. The Environmental Protection Agency regulates watersafety in municipal water supplies and community wells but not in private wells, which may lead to somerisks. Risks for children from drinking contaminated water are greater than for adults, in many cases dueto the dose response and dose per body weight of both microorganisms and chemical contaminants. The American Academy of Pediatrics has published a policy statement that may assist practitionersin providing advice to families about proper construction, maintenance, and surveillance of wells toprovide a safe drinking water supply, including the recommended periodicity and costs of routine testingof well water. Annual testing for fecal coliforms is recommended; for most chemical contaminants suchas hydrocarbons, testing should be performed every 3 to 5 years. More frequent testing may beindicated if there are outbreaks of intestinal disease in the community. For the family described in the vignette, the source of their water is important to discuss. Because thefamily's water supply is from their own private well, they need to evaluate the water supply to determineif it is safe. Meanwhile, they should use boiled water. Use of bottled water, including distilled, isexpensive and may not contain fluoride. In addition, bottled water may contain chemical contaminantsfrom the plastic that could confer long-term health risks for the family. Although infections in children caused by drinking water that is contaminated by fecal coliforms orparasites (especially Cryptosporidium) may be identified by gastrointestinal symptoms such as diarrheaand vomiting, chronic ingestion of chemical contaminants may result in "silent intoxication" or morevague symptom complexes. There is evidence that industrial hydrocarbons such as trichloroethylenes,used as degreasers and aerosolized solvents, can seep into groundwater, reservoirs, and the watersupply. These agents have been linked to both low birthweight and birth defects. Filtered tap water (via faucet or pitcher/carafe-mounted filters) may reduce the presence of lead,Cryptosporidium, and Giardia lamblia, but they are meant for water that is already processed through amunicipal water supply and are not intended to detoxify heavily contaminated water. Families who have private wells should be advised to engage the services of a professional wellmonitoring technician available through local water, health, or agricultural departments or private wellinstallation and service firms. Those who choose to self-monitor and maintain wells may find resourcesfor acceptable levels of fecal coliform contaminants and instructions for decontamination (usinghousehold bleach) from local health departments. Infant formula should not be prepared with well water containing high concentrations of nitrates (>10mg/dL), and parents should be made aware of other potential ground water contaminants that may havelong-term effects on health for both children and adults, including arsenic, methyl tertiary butyl ester, andperchlorates.

SUGGESTED

Dr_Faqehi

Page 49: 04 Poisoning

American Academy of Pediatrics 2012 PREP SA on CD-ROM

Copyright 2012 © American Academy of Pediatrics 24

READING:

American Academy of Pediatrics. Cryptosporidiosis. In: Pickering LK, Baker CJ, Kimberlin DW, LongSS, eds. Red Book: 2009 Report of the Committee on Infectious Diseases. 28th ed. Elk Grove Village,IL: American Academy of Pediatrics; 2009:272-273. Accessed January 2011 at:http://aapredbook.aappublications.org/cgi/content/full/2009/1/3.32

Rogan WJ, Brady MT, the Committee on Environmental Health, and the Committee on InfectiousDiseases. Drinking water from private wells and risks to children. Pediatrics. 2009;123: e1123-e1137.DOI: 10.1542/peds.2009-0752. Accessed January 2011 at::http://pediatrics.aappublications.org/cgi/content/full/123/6/e1123

United States Environmental Protection Agency. Drinking Water Contaminants: National PrimaryDrinking Water Regulations. 2010. Accessed January 2011 at:http://water.epa.gov/drink/contaminants/index.cfm

Watson RE, Jacobson CF, Williams AL, Howard WB, DeSesso JM. Trichloroethylene-contaminateddrinking water and congenital heart defects: a critical analysis of the literature. Reprod Toxicol.2006;21:117-147. DOI: 10.1016/j.reprotox.2005.07.013. Abstract accessed January 2011 at:http://www.ncbi.nlm.nih.gov/pubmed/16181768

Dr_Faqehi

Page 50: 04 Poisoning

American Academy of Pediatrics 2012 PREP SA on CD-ROM

Copyright 2012 © American Academy of Pediatrics 25

Question: 246

A 16-month-old girl is brought to the emergency department because of persistent crying for severalhours. She has had clear rhinorrhea, cough, and an undocumented fever for 3 days. She has hadadequate oral intake, has vomited twice, and has had no diarrhea. She has no underlying medicalconditions and no exposures. Current medications include acetaminophen and an over-the-countercough and cold medicine. Physical examination reveals an agitated, crying child, who is inconsolable.Her temperature is 37.8°C, heart rate is 192 beats/min, respiratory rate is 36 breaths/min, bloodpressure is 122/78 mm Hg, and oxygen saturation is 98%. The remainder of physical examinationfindings are normal. Complete blood count, electrolytes, urinalysis, and cerebrospinal fluid studies yieldnormal results. Electrocardiography shows sinus tachycardia.

Of the following, the MOST likely cause of this child's symptoms is

A. acetaminophen overdose

B. early meningitis

C. intussusception

D. myocarditis

E. reaction to cold and cough medicine

Dr_Faqehi

Page 51: 04 Poisoning

American Academy of Pediatrics 2012 PREP SA on CD-ROM

Copyright 2012 © American Academy of Pediatrics 26

Preferred Response: ECritique: 246

The differential diagnosis for an irritable and crying child, such as the girl described in the vignette, isbroad, ranging from trauma to infection to metabolic conditions to toxicologic issues. This girl'sevaluation has ruled out many of the serious entities in the differential diagnosis, such as meningitis. Infact, her history and abnormal vital signs suggest a likely cause: toxic effects from cough and coldmedication. Over-the-counter cough and cold medications may contain one or more components, usually withdecongestant, antihistamine, expectorant, or cough suppressant effects. Multiple studies have shownthat these medications have little or no efficacy in the treatment of upper respiratory tract infections inchildren compared with placebo. For this reason, the United States Food and Drug Administration (FDA)was petitioned and convened a meeting in 2007 to review the safety of these products. Before themeeting, however, major manufacturers pulled from production products marketed for children youngerthan 2 years of age and added a label warning against using the medications for sedative purposes.Although experts further recommended that the products be eliminated for children younger than 6 yearsof age, a pharmaceutical trade group subsequently voluntarily changed labeling to warn about their usein children younger than 4 years. The most common decongestant in cold and cough preparations is pseudoephedrine, which mayhave sympathomimetic effects, such as tachycardia, arrhythmia, hypertension, central nervous systemstimulation, and rarely seizures. Antihistamines such as diphenhydramine, brompheniramine,chlorpheniramine, and carbinoxamine can lead to central nervous system agitation or depression,dysrhythmias, hypertension, seizures, and in extreme cases, respiratory depression. Guaifenesin, themost commonly used expectorant, is generally well tolerated but may cause mild gastrointestinaldiscomfort. Dextromethorphan and codeine are the most widely used cough suppressants and appear tohave efficacy greater than placebo in adults, but such efficacy is not noted for children.Dextromethorphan is derived from opiates and acts at the central nervous system level to inhibit cough.Although it does not have the addictive or analgesic properties of other opiates, it can cause euphoria,hallucinations, lethargy, coma, nausea, dizziness, drowsiness, ataxia, nystagmus, and urinary retention.It does have abuse potential because of the euphoric effects it may produce. In lower doses, codeinecan lead to somnolence, ataxia, gastrointestinal distress, and pruritus; in higher doses, it leads todepressed mental status and respiratory depression. Since 1997, the American Academy of Pediatricshas recommended against the use of codeine- and dextromethorphan-containing cough remedies inchildren. Recently, the use of cold and cough preparations has been implicated in the deaths of anumber of infants and children, further strengthening the argument against their use. The absence of abnormalities in the cerebrospinal fluid of this girl makes meningitis very unlikely,even at an early stage. Intussusception causes episodes of crying in children, but they are typicallyintermittent rather than continuous. Alternatively, intussusception may present with lethargy; continuousagitation is not typical. Myocarditis may result in tachycardia out of proportion to fever, but usualelectrocardiographic findings, including decreased precordial voltages, are absent in this patient.Because some cold and cough preparations may contain analgesics, it is prudent to measure the serumconcentration of acetaminophen for this patient, but agitation, tachycardia, and hypertension are notcharacteristic of

Dr_Faqehi

Page 52: 04 Poisoning

American Academy of Pediatrics 2012 PREP SA on CD-ROM

Copyright 2012 © American Academy of Pediatrics 27

acetaminophen overdose.

SUGGESTED READING:

Carr BC. Efficacy, abuse, and toxicity of over-the-counter cough and cold medicines in the pediatricpopulation. Curr Opin Pediatr. 2006;18:184-188. DOI: 10.1097/01.mop.0000193274.54742.a1. Abstractaccessed January 2011 at: http://www.ncbi.nlm.nih.gov/pubmed/16601501

Committee on Drugs. American Academy of Pediatrics. Use of codeine- and dextromethorphan-containing cough remedies in children. Pediatrics. 1997;99:918-920. DOI: 10.1542/peds.99.6.918.Accessed January 2011 at: http://pediatrics.aappublications.org/cgi/content/full/99/6/918

Gunn VL, Taha SH, Liebelt EL, Serwint JR. Toxicity of over-the-counter cough and cold medications.Pediatrics. 2001;108:e52. Accessed January 2011 at:http://pediatrics.aappublications.org/cgi/content/full/108/3/e52

Kuehn BM. Debate continues over the safety of cold and cough medicines for children. JAMA.2008;300:2354-2356

Rimsza ME, Newberry S. Unexpected infant deaths associated with use of cough and coldmedications. Pediatrics. 2008;122:e318-e322. DOI: 10.1542/peds.2007-3813. Accessed January 2011at:: http://pediatrics.aappublications.org/cgi/content/full/122/2/e318

Sharfstein JM, North M, Serwint JR. Over the counter but no longer under the radar - pediatric coughand cold medications. N Engl J Med. 2007;357:2321-2324. Accessed January 2011 at:http://www.nejm.org/doi/full/10.1056/NEJMp0707400

Dr_Faqehi

Page 53: 04 Poisoning

2011 PREP SA on CD-ROM

Question: 161

You receive a call from the emergency department at 11:00 pm on New Year’s Eve. Five patientshave presented with foreign body ingestions.

Of the following, the foreign body that is MOST appropriate to be removed first is a

A. 1999 penny in the stomach of an 18-month-old

B. AAA battery in the stomach of a 4-year-old

C. button battery in the mid-esophagus of a 2-year-old

D. quarter in the esophagus of a 3-year-old who vomited twice and is refusing solids

E. toothbrush in the stomach of an asymptomatic 16-year-old, which she swallowed 2 hours ago

Copyright © 2010 by the American Academy of Pediatrics page 1

Dr_Faqehi

Page 54: 04 Poisoning

2011 PREP SA on CD-ROM

Preferred Response: CCritique: 161

Management of ingested foreign bodies depends on the item ingested, its anatomic location,and the presence of symptoms. In most cases, gastric foreign bodies may be managedconservatively, but esophageal impactions require urgent removal. Button batteries present aunique problem because they contain toxic heavy metals as well as alkaline compounds (eg,sodium and potassium hydroxide) that are caustic to esophageal mucosa. Significantesophageal injury (including perforation) has been reported from button batteries lodged in theesophagus for as few as 6 hours. Most complications are caused by larger batteries (20 to 23mm diameter), although significant esophageal injury has been reported with batteries as smallas 8 mm in diameter. Symptoms of dysphagia (including feeding refusal, excessive drooling,difficulty swallowing) or emesis suggest esophageal impaction.

Regardless of the presence or absence of symptoms, a radiograph of the neck, chest, andabdomen should be obtained in all patients who present with a history of possible batteryingestion (Item C161A). Button batteries lodged in the esophagus must be removedendoscopically as soon as possible after detection. Batteries (both button and cylindrical)detected in the stomach usually traverse the gastrointestinal tract without incident, with morethan 80% passing within 48 hours. In asymptomatic patients, battery location should beassessed radiographically at that time, and if the object has not passed the pylorus, removal isrecommended. In symptomatic patients, urgent endoscopic removal is indicated.

In the United States, foreign body ingestions are primarily a pediatric problem, with morethan 80% of reported cases occurring in children. The exception is esophageal meat impaction,which is the most common foreign body ingestion-related problem in adults who do not haveunderlying psychiatric disturbances. Although the precise incidence is unknown, more than100,000 cases of foreign body ingestion in children were reported to the American Associationof Poison Control Centers toxic exposure surveillance system in 2000. Of these, 98% wereunintentional. Coins are the most commonly reported foreign bodies, and most traverse thegastrointestinal tract without difficulty. Toy parts, sharp objects (needles, pins), batteries,chicken or fish bones, and food impactions are other frequently reported items.

A coin lodged in the esophagus must be removed emergently if the patient is unable tohandle secretions (Item C161B). Otherwise, endoscopy may be carried out within 12 to 24hours, by which time up to 30% of coins (mostly those in the distal third of the esophagus) willhave passed into the stomach. In all instances of esophageal and gastric foreign bodies, a chestradiograph should be performed immediately prior to endoscopy to confirm location anddetermine whether the object has passed the lower esophageal sphincter or the pyloric outlet.Coins in the stomach require no immediate therapy in asymptomatic patients. Parents should beinstructed to examine stools for coin passage, and if the coin is not retrieved, a follow-upradiograph may be obtained in 2 to 3 weeks. Only then, if the coin is retained in the stomach,should removal be considered. Of note, the composition of pennies changed from copper topredominantly zinc in 1982. Although corrosion by gastric acid may release absorbable zincchloride, no difference in the management of ingested pennies is recommended in asymptomaticpatients. Toothbrushes and other long objects (eg, tongue depressors) have been reported asunintentional ingestions in adolescents who have bulimia. The objects are swallowed whileusing them to induce vomiting and require endoscopic removal.

Copyright © 2010 by the American Academy of Pediatrics page 2

Dr_Faqehi

Page 55: 04 Poisoning

2011 PREP SA on CD-ROM

Suggested reading:

Arana A, Hauser B, Hachimi-Idrissi S, Vandenplas Y. Management of ingested foreign bodies inchildhood and review of the literature. Eur J Pediatr. 2001;160:468-472. DOI:10.1007/s004310100788. Abstract available at: http://www.ncbi.nlm.nih.gov/pubmed/11548183

Litovitz T, Schmitz BF. Ingestion of cylindrical and button batteries: an analysis of 2382 cases.Pediatrics. 1992;89: 747-757. Abstract available at:http://pediatrics.aappublications.org/cgi/content/abstract/89/4/747

Mas E, Olives J-P. Toxic and traumatic injury of the esophagus. In: Kleinman RE, Goulet O, Mieli-Vergani G, Sanderson I, Sherman P, Shneider B, eds. Walker’s Pediatric GastrointestinalDisease. 5th ed. Hamilton, Ontario, Canada: BC Decker; 2008:105-116

Rebhandl W, Steffan I, Schramel P, et al. Release of toxic metals from button batteries retained inthe stomach: an in vitro study. J Pediatr Surg. 2002;37:87-92. DOI: 10.1053/jpsu.2002.29435.Abstract available at: http://www.ncbi.nlm.nih.gov/pubmed/11781994

Yardeni D, Yardeni H, Coran AG, Golladay ES. Severe esophageal damage due to button batteryingestion: can it be prevented? Pediatr Surg Int. 2004;20:496-501. DOI: 10.1007/s00383-004-1223-6. Available at: http://springerlink.metapress.com/content/rdrxu127rlw6twgb/fulltext.html

Copyright © 2010 by the American Academy of Pediatrics page 3

Dr_Faqehi

Page 56: 04 Poisoning

2011 PREP SA on CD-ROM

Critique: 161

(Courtesy of D Mulvihill)A 2-year-old child who ingested a watch battery that is now located in mid esophagus. Althoughthe foreign body resembles a coin, the more radio-opaque peripheral rim is not a feature of coins.

Copyright © 2010 by the American Academy of Pediatrics page 4

Dr_Faqehi

Page 57: 04 Poisoning

2011 PREP SA on CD-ROM

Critique: 161

(Courtesy of D Mulvihill)Coin in the upper esophagus of a child who had been coughing for weeks. There is widening ofthe esophageal wall.

Copyright © 2010 by the American Academy of Pediatrics page 5

Dr_Faqehi

Page 58: 04 Poisoning

2011 PREP SA on CD-ROM

Question: 177

The father of a 4-year-old boy calls you from the emergency department near the family’svacation home in another state. His son just swallowed a quarter, and an abdominal film shows itto be in the gastric fundus.

Of the following, you are MOST likely to recommend

A. a follow-up abdominal radiograph in 2 weeks

B. a short course of metoclopramide

C. endoscopic removal of the coin

D. no further follow-up because the coin will pass unaided

E. polyethylene glycol 3550 as an osmotic cathartic

Copyright © 2010 by the American Academy of Pediatrics page 6

Dr_Faqehi

Page 59: 04 Poisoning

2011 PREP SA on CD-ROM

Preferred Response: ACritique: 177

Coins are the most common foreign bodies ingested by children in the United States and areresponsible for approximately 25,000 emergency department visits annually. However, becausemany coin ingestions are not witnessed and most are asymptomatic, a significant number likelyare unreported, and others may be managed at home. Most swallowed coins traverse thegastrointestinal tract unaided and without incident. Despite the fact that most patients do notpresent with symptoms, a radiograph should be taken of the neck, chest, and abdomen toconfirm the coin ingestion and estimate its size and location, as described for the boy in thevignette. For an asymptomatic child who has an ingested coin located in the stomach (or smallbowel), no further immediate management is required. Parents should be instructed to examinethe child’s stools because most coins pass within 4 to 6 days. However, if nothing is recovered,a follow-up radiograph at 2 weeks should confirm coin passage versus retention and guidefurther therapy. Interestingly, up to 40% of excreted coins are missed by parents, and follow-upradiographs are negative.

Because most ingested coins pass spontaneously and are not associated with symptoms,the necessity of an initial radiograph has been questioned. However, some coins may lodge inthe esophagus (Item C177). Most esophageal foreign bodies cause immediate problems withhandling of secretions, feeding refusal, dysphagia, or a sensation of something in the chest,indicating the need for urgent endoscopic removal. Coins located in the distal third of theesophagus, however, may not precipitate acute symptoms. Nevertheless, retained esophagealforeign bodies are responsible for significant morbidity, including esophageal ulceration andperforation. A coin also is more likely to lodge in the esophagus of a patient who has underlyingesophageal pathology (eg, previous surgery, esophageal stricture, fundoplication, eosinophilicesophagitis). Accordingly, a radiograph is recommended in all cases.

Because of its size, a quarter may be retained in the stomach, especially in toddlers. In theabsence of symptoms, it can be removed safely via endoscopy after a few weeks if a follow-up radiograph indicates gastric retention. Smaller coins also may remain in the stomach,particularly when multiple coins are ingested and among patients who have underlying motilitydisturbances or a history of prior surgery for pyloric stenosis. Patients who present with ordevelop symptoms should undergo endoscopy within 12 to 24 hours.

Because the composition of pennies changed from copper to predominantly zinc in 1982,concerns have been raised about the potential toxicity of zinc chloride released from thecorrosive action of gastric acid following penny ingestion. However, no difference in themanagement of ingested pennies from that of other coins currently is recommended inasymptomatic patients.

Alternate or adjunct therapeutic options following coin ingestion are not recommended.Drinking carbonated beverages to dilate the esophagus and facilitate passage of esophagealcoins (previously studied in adult subjects), parenteral administration of glucagon, treatment withprokinetic agents to enhance gastrointestinal motility, and the use of cathartics have not provedto be effective.

Suggested reading:

Copyright © 2010 by the American Academy of Pediatrics page 7

Dr_Faqehi

Page 60: 04 Poisoning

2011 PREP SA on CD-ROM

Arana A, Hauser B, Hachimi-Idrissi S, Vandenplas Y. Management of ingested foreign bodies inchildhood and review of the literature. Eur J Pediatr. 2001;160:468-472. DOI:10.1007/s004310100788. Abstract available at: http://www.ncbi.nlm.nih.gov/pubmed/11548183

Conners GP. Management of asymptomatic coin ingestion. Pediatrics. 2005;116:752-753. DOI:10.1542/peds.2005-0062. Available at:http://pediatrics.aappublications.org/cgi/content/full/116/3/752

Wahbeh G, Wyllie R, Kay M. Foreign body ingestion in infants and children: location, location,location. Clin Pediatr (Phila). 2002;41:633-640

Copyright © 2010 by the American Academy of Pediatrics page 8

Dr_Faqehi

Page 61: 04 Poisoning

2011 PREP SA on CD-ROM

Critique: 177

(Courtesy of M Wright)Coin in the upper esophagus of a child who had been coughing for weeks.

Copyright © 2010 by the American Academy of Pediatrics page 9

Dr_Faqehi

Page 62: 04 Poisoning

2011 PREP SA on CD-ROM

Question: 193

You are called to the emergency department to see a 2 1/2-year-old boy at 9 pm on New Year'sEve. His parents found him crying on the kitchen floor with an open, spilled container of granulardrain cleaner beside him. On physical examination, the boy is alert and screaming. His lips areerythematous and edematous, but he can handle his secretions without difficulty.

Of the following, the MOST appropriate next step is

A. administration of broad-spectrum antibiotics

B. administration of intravenous corticosteroids

C. barium swallow study

D. observation in the hospital for 24 hours

E. upper gastrointestinal endoscopy

Copyright © 2010 by the American Academy of Pediatrics page 10

Dr_Faqehi

Page 63: 04 Poisoning

2011 PREP SA on CD-ROM

Preferred Response: ECritique: 193

More than 200,000 known exposures to household cleaning agents are reported annually inthe United States. Caustic ingestions represent an unknown fraction of these exposures,although ingestions occur most commonly in children younger than 6 years of age, with mostaffected children being 1 to 4 years old. The granular drain cleaner to which the toddlerdescribed in the vignette has been exposed typically is formulated as a strong base (oftensodium hydroxide) with a pH of greater than 11.5. Despite evidence of oral burns (erythematous,swollen lips), the boy apparently can handle secretions without difficulty. Nevertheless, anypatient who has signs of oral injury or symptoms of odynophagia/dysphagia (drooling, choking,solid food refusal) following a known or suspected caustic ingestion should undergo uppergastrointestinal endoscopy within 12 to 24 hours.

Acids and alkalis, including oven, toilet bowl, tile, and drain cleaners, comprise 50% ofingested caustic agents; bleaches (30% to 40%) and laundry detergents (10% to 20%)comprise the remainder. Fortunately, because of the use of childproof containers and theenactment of laws to limit the concentration of household cleaning products, the overallincidence of caustic ingestions in children appears to be declining. Nevertheless, these agentsare responsible for significant morbidity in young children and are associated with a highmortality rate in adolescents and adults, in whom the ingestion most often represents a suicideattempt.

Among pediatric patients, alkalis are ingested more commonly than acids and account formost postingestion complications. Strong acids, which account for approximately 15% ofingestions in children, cause immediate oral pain and have a bitter taste, thus limiting the quantityof ingestion. These agents have a low viscosity and, therefore, undergo rapid transit to thestomach, where the caustic injury is most severe. In fact, the esophagus is spared in 80% ofcases. Alkalis usually are odorless and tasteless, even after several swallows. As aconsequence, alkaline products are likely to be ingested in greater amounts. Alkali causes aliquefaction necrosis rather than the coagulative necrosis from strong acids. The esophagus isthe site of greatest mucosal damage following alkali consumption, although gastric injury alsomay occur, and the degree of injury is related to both the amount and concentration of thematerial ingested. Products that have a pH greater than 11 (drain cleaner typically has a pH ofgreater than 11.5) are associated with a high risk of caustic burns, and granular products causea higher rate of injury compared with liquids.

Postingestion evaluation and management depend on the nature of the ingestion and thepresence or absence of symptoms. Standard laundry bleach probably is the single mostcommonly ingested household cleaning product. This agent contains 5% sodium hypochloritebuffered to a pH of less than 11, and patients rarely require a diagnostic evaluation or specifictherapy following exposure. In contrast, endoscopic evaluation is indicated following theingestion of known caustic agents irrespective of the degree of symptoms. In some cases, theingested product may be swallowed rapidly (particularly with an intentional ingestion), and earlysymptoms may be absent. Endoscopy undertaken before 12 hours after the incident may fail toidentify the severity and extent of injury, and delay of endoscopy more than 24 hours after theincident can increase the risk of esophageal perforation.

Asymptomatic patients who present with a questionable history of corrosive ingestion may

Copyright © 2010 by the American Academy of Pediatrics page 11

Dr_Faqehi

Page 64: 04 Poisoning

2011 PREP SA on CD-ROM

be observed and allowed to consume liquids. If symptoms of dysphagia develop, a bariumswallow study should be performed to assess for esophageal stricture. However, a bariumswallow study is not helpful early postingestion in symptomatic patients, such as the childdescribed in the vignette, because it cannot be used to determine the nature or extent ofesophageal injury.

Endoscopy permits grading of the injury and guides further management. Caustic injury tothe esophagus is graded on a scale of 0 to 3. Patients who have grades 0 and 1 injuries (grade1 is seen in 80% of patients who have an endoscopically confirmed injury) may be sent homeafter the procedure if they are tolerating feedings. Patients who have grades 2 to 3 lesions areat greater risk for both short- and long-term complications, including bleeding and esophagealstrictures. Those who have grade 2 injuries require intravenous support until the extent ofdamage becomes evident. They may be fed postendoscopy, as long as no signs of perforationare noted. Grade 3 injuries, characterized by circumferential mucosal necrosis, pose thegreatest threat of early perforation and fistula formation and require long-term parenteral supportfor administration of fluids and nutrition. Grade 2 lesions are associated with a greater than 10%likelihood of subsequent esophageal stricture; stricture formation is likely with grade 3 lesions.

Pharmacologic management approaches to caustic injuries are either controversial(corticosteroids) or have failed to undergo controlled clinical trials (antibiotics). Corticosteroidsmay be used in patients who have airway symptoms related to perilaryngeal edema, althoughtheir efficacy has not been fully established. Antibiotics, when prescribed, should be reservedfor patients who have grades 2 or 3 injuries, in cases where steroids are administered, and inthe setting of suspected or documented perforation.

Suggested reading:

Betalli P, Falchetti D, Giuliani S, et al. Caustic ingestion in children: is endoscopy alwaysindicated? The results of an Italian multicenter observational study. Gastrointest Endosc.2008;68:434-439. DOI: 10.1016/j.gie.2008.02.016. Abstract available at:http://www.ncbi.nlm.nih.gov/pubmed/18448103

Kay M, Wyllie R. Caustic ingestions in children. Curr Opin Pediatr. 2009;21:651-654. Abstractavailable at: http://www.ncbi.nlm.nih.gov/pubmed/19543088

Kay M, Wyllie R. Symptoms may not adequately predict extent of injury in pediatric patients aftera caustic ingestion [letter]. Gastrointest Endosc. 2009;69:1407. DOI: 10.1016/j.gie.2008.10.003.Available at: http://www.giejournal.org/article/PIIS0016510708027041/fulltext

Lamireau T, Rebouissoux L, Denis D, Lancelin F, Vergnes P, Fayon M. Accidental causticingestion in children: is endoscopy always mandatory? J Pediatr Gastroenterol Nutr.2001;33:81-84. Available at:http://journals.lww.com/jpgn/Fulltext/2001/07000/Accidental_Caustic_Ingestion_in_Children__Is.14.aspx

Copyright © 2010 by the American Academy of Pediatrics page 12

Dr_Faqehi

Page 65: 04 Poisoning

2011 PREP SA on CD-ROM

Turner A, Robinson P. Respiratory and gastrointestinal complications of caustic ingestion inchildren. Emerg Med J. 2005;22:359—361.

Wilsey MJ Jr, Scheimann AO, Gilger MA. The role of upper gastrointestinal endoscopy in thediagnosis and treatment of caustic ingestion, esophageal strictures, and achalasia in children.Gastrointest Endosc Clin N Am. 2001;11:767-787. Abstract available at:http://www.ncbi.nlm.nih.gov/pubmed/11689365

Copyright © 2010 by the American Academy of Pediatrics page 13

Dr_Faqehi

Page 66: 04 Poisoning

2011 PREP SA on CD-ROM

Question: 208

A 2-year-old boy is brought to the emergency department after his father found him with the leaffrom a foxglove plant in his mouth (Item Q208). He has had one episode of emesis and iscomplaining of abdominal pain. On physical examination, his heart rate is 140 beats/min,respiratory rate is 24 breaths/min, blood pressure is 100/60 mm Hg, and oxygen saturation is100%. His pupils are 4 mm and briskly reactive to 2 mm. The remainder of his examination findingsare normal.

After administering activated charcoal, the MOST appropriate next step is

A. abdominal radiography

B. electrocardiography

C. serum creatine phosphokinase assessment

D. serum sodium assessment

E. urine toxicology screening

Copyright © 2010 by the American Academy of Pediatrics page 14

Dr_Faqehi

Page 67: 04 Poisoning

2011 PREP SA on CD-ROM

Question: 208

(Courtesy of M Wright)Foxglove

Copyright © 2010 by the American Academy of Pediatrics page 15

Dr_Faqehi

Page 68: 04 Poisoning

2011 PREP SA on CD-ROM

Preferred Response: BCritique: 208

More than 100,000 plant exposures are reported to the American Association of PoisonControl Centers annually, most of which are not serious. Although there are many potentiallytoxic plants in the environment, only a few account for most of the fatalities and emergencydepartment visits. In addition, most of these occur in adults who intentionally ingest the plant, useit for perceived medicinal purposes, or mistake the plant for a nontoxic look-alike.

In general, the toxicities of plants affect one or more of three organ systems: cardiac,neurologic, or gastrointestinal (Item C208). As with most poisonings, evaluation and treatmentare directed at decontamination and assessment/support of vital functions. Plant identification ishelpful in determining a treatment plan (and can be facilitated by the local Poison Control Center),but it should not delay the initial evaluation and decontamination.

Because most serious plant ingestions have cardiac effects, electrocardiography should beconsidered in affected patients. This is especially true for the patient described in the vignette,who has ingested foxglove, a source of potent cardiac glycosides. Abdominal radiography,serum sodium determination, and urine toxicology screening are of no utility. Rhabdomyolysismay be a late finding in foxglove ingestion. A serum creatine phosphokinase determination maybe considered if the patient develops muscle tenderness or myoglobinuria.

Suggested reading:

Franklin RL, Rodgers GB. Unintentional child poisonings treated in United States hospitalemergency departments: national estimates of incident cases, population-based poisoning rates,and product involvement. Pediatrics. 2008;122:1244-1251. DOI: 10.1542/peds.2007-3551.Available at: http://pediatrics.aappublications.org/cgi/content/full/122/6/1244

Froberg B, Ibrahim D, Furbee RB. Plant poisoning. Emerg Med Clin North Am. 2007;25:375-433.DOI: 10.1016/j.emc.2007.02.013. Abstract available at:http://www.ncbi.nlm.nih.gov/pubmed/17482026

Copyright © 2010 by the American Academy of Pediatrics page 16

Dr_Faqehi

Page 69: 04 Poisoning

2011 PREP SA on CD-ROM

Critique: 208

Copyright © 2010 by the American Academy of Pediatrics page 17

Dr_Faqehi

Page 70: 04 Poisoning

2010 PREP SA on CD-ROM

Question: 32

The parents of a 12-year-old boy bring him to the emergency department after finding himunresponsive in bed when they tried to wake him for school. They report that he has had norecent illnesses and was in his usual state of health when he went to bed last night. Of note, hehas enuresis, treated with imipramine. On physical examination, he is responsive only to pain,his heart rate is 120 beats/min, respiratory rate is 6 breaths/min, and blood pressure is 60/40 mmHg. His pupils are 6 mm, equal, and sluggishly reactive. All other findings are within normalparameters. He is endotracheally intubated, ventilated with 100% oxygen, placed on a cardiacmonitor, and given a 20-mL/kg bolus of normal saline. Electrocardiography demonstrates sinustachycardia, PR interval of 130 msec, and QRS duration of 140 msec.

Of the following, the next MOST appropriate step is to

A. administer adenosine

B. administer amiodarone

C. administer sodium bicarbonate

D. begin external pacing

E. perform synchronized cardioversion at 0.5 J/kg

Copyright © 2010 by the American Academy of Pediatrics page 1

Dr_Faqehi

Page 71: 04 Poisoning

2010 PREP SA on CD-ROM

Preferred Response: CCritique: 32

The coma, respiratory depression, tachycardia, hypotension, and dilated pupils describedfor the patient in the vignette are consistent with an acute tricyclic antidepressant (TCA)overdose. Among the additional signs that might be observed are seizures, dysrhythmias, andother anticholinergic features such as dry mouth, hyperthermia, urinary retention, flushed skin,and agitation. Tricyclic antidepressants cause these clinical features by inhibiting a variety ofneurotransmitter receptors, including muscarinic acetylcholine, alpha-1-adrenergic, gammaaminobutyric acid (GABA), and histamine receptors as well as cardiac fast sodium channels.

The most serious toxicities from TCA overdose affect the heart and central nervous system(CNS). Because of the TCA effect on fast sodium channels, conduction velocity is decreased. Inaddition, repolarization duration and absolute refractory periods are prolonged. These effects,coupled with alpha-1-adrenergic antagonism, are responsible for the hypotension andconduction delays that are seen commonly. CNS excitation and seizures or depression may berelated to effects on GABA or histamine receptors.

Initial management of a TCA overdose begins with ensuring a patent airway and restoringadequate oxygenation, ventilation, and perfusion. Intubation and mechanical ventilation often arenecessary, as is fluid resuscitation with boluses of normal saline. Seizures are treated withbenzodiazepines. Alpha-adrenergic pressors (eg, norepinephrine) may be required to treatrefractory hypotension. Decontamination should be performed with activated charcoal. Althoughacetaminophen and aspirin concentrations should be measured, especially in intentionalingestions to evaluate for possible coingestants, measurement of TCA concentrations is notclinically useful.

The single most useful diagnostic and prognostic test in the setting of a TCA overdose iselectrocardiography. In a study from 1985, toxicologists found that a QRS duration of greaterthan 100 msec predicted seizures in 34% and dysrhythmias in 14% of patients who had TCAoverdoses. The QRS widening is related to fast sodium channel blockade caused by direct TCAeffects and exacerbated by acidemia. These effects can be overcome by the administration ofsodium bicarbonate boluses. Sodium bicarbonate should be administered until the QRS durationis less than 100 msec. The exact mechanism for this effect is unknown.

Adenosine and synchronized cardioversion are treatments for supraventricular tachycardia.Amiodarone is a drug of choice for ventricular arrhythmias. External pacing is appropriatetreatment for refractory, symptomatic bradycardia.

References:

Boehnert MT, Lovejoy FH Jr. Value of QRS duration versus the serum drug level in predictingseizures and ventricular arrhythmias after an acute overdose of tricyclic antidepressants. NEngl J Med. 1985;313:474-479. Abstract available at:http://www.ncbi.nlm.nih.gov/pubmed/4022081

Hutchinson MD, Traub SJ. Tricyclic antidepressant poisoning. UpToDate Online 16.3. 2008.Available at:http://www.utdol.com/online/content/topic.do?topicKey=ad_tox/10025&selectedTitle=1~150&sou

Copyright © 2010 by the American Academy of Pediatrics page 2

Dr_Faqehi

Page 72: 04 Poisoning

2010 PREP SA on CD-ROM

rce=search_result11

Jacob J. Toxicity, antidepressant. In: eMedicine Specialties, Emergency medicine, Toxicology.2008. Available at: http://www.emedicine.com/emerg/topic37.htm

Woolf AD, Erdman AR, Nelson LS, et al. Tricyclic antidepressant poisoning: an evidence-basedconsensus guideline for out-of-hospital management. Clin Toxicol (Phila). 2007;45:203-233.Brief summary available at:http://guidelines.gov/summary/summary.aspx?doc_id=9906&nbr=005302&string=tricyclic+AND+antidepressant+AND+overdose

Copyright © 2010 by the American Academy of Pediatrics page 3

Dr_Faqehi

Page 73: 04 Poisoning

2010 PREP SA on CD-ROM

Question: 48

The mother of one of your patients calls frantically because she just found her 2-year-olddaughter with an open bottle of prenatal vitamins and several of the tablets in her mouth. Thechild is acting normally. The mother reports that the label says there is 30 mg of elemental ironper tablet and five tablets are missing from the bottle she picked up at the pharmacy thismorning. Her daughter weighs 25 lb.

Of the following, the MOST appropriate advice to give the mother is to

A. bring the child to office in the morning for assessment of serum iron concentration

B. give the child activated charcoal

C. give the child syrup of ipecac

D. observe the child at home for symptoms

E. take the child to the nearest emergency department

Copyright © 2010 by the American Academy of Pediatrics page 4

Dr_Faqehi

Page 74: 04 Poisoning

2010 PREP SA on CD-ROM

Preferred Response: DCritique: 48

Iron overdoses continue to be common among children younger than 6 years of age. Most ofthese ingestions are unintentional and do not result in significant toxicity. However, iron ingestioncan cause fatalities, especially in the setting of an intentional ingestion or exposure to an adultpreparation.

Management of iron ingestion begins with determining, if possible, how much elemental ironwas ingested and if the patient is symptomatic. Any child who is symptomatic within 6 hoursafter ingesting iron, regardless of the estimated dose, or if the dose is unknown, should bebrought to medical attention. In the asymptomatic child, ingestions of less than 40 mg/kg ofelemental iron are not significant, and the patient can be observed at home. The mother in thevignette can be reassured that because her child ingested 13.2 mg/kg of elemental iron (<40mg/kg), she is unlikely to develop toxicity and can be observed at home. If she remainsasymptomatic for more than 6 hours, no further evaluation or treatment is necessary. If shedevelops symptoms, she should be taken to an emergency department for evaluation andtreatment.

For the patient who has a significant ingestion, laboratory evaluation should includemeasurement of a serum iron concentration within 4 hours of ingestion, serum electrolyte andaminotransferase determinations, a complete blood count, and coagulation tests. Laboratoryindicators of a potentially significant ingestion include serum iron concentration greater than 350

mcg/dL (62.7 mcmol/L), white blood cell count greater than 15.0x103/mcL (15.0x109/L), andserum glucose values greater than 150 mg/dL (8.3 mmol/L). In a symptomatic patient, abdominalradiography should be used to look for the presence of retained tablets in the gastrointestinaltract. If present, gastrointestinal decontamination using whole-bowel irrigation is indicated.Patients who have severe symptoms, anion gap acidosis, serum iron concentrations of greaterthan 500 mcg/dL (89.5 mcmol/L), or a significant number of pills visible on abdominal radiographyshould be treated with deferoxamine to chelate free circulating iron. Neither activated charcoal,which adsorbs iron poorly, nor syrup of ipecac is indicated for decontamination.

Iron is both a corrosive and a cellular toxin. The clinical phases of significant iron toxicity areattributable directly to these two mechanisms. Phase 1 (gastrointestinal phase) occurs between30 minutes to 6 hours after ingestion and includes vomiting, diarrhea, abdominal pain, andhematemesis or melena. Such signs and symptoms are caused by the agent’s corrosive effectson the gastrointestinal mucosa. Phase 2 (latent phase) occurs 6 to 12 hours after ingestion butcan last as long as 24 hours. Patients often are asymptomatic during this time while free iron istaken up into the reticuloendothelial organs. Phase 3 (shock, metabolic acidosis, hepatic failurephase) may be seen as early as 6 to 12 hours after ingestion and is the result of mitochondrialdysfunction and cell death. Phase 4 (bowel obstruction phase) occurs 2 to 8 weeks after theacute ingestion and results from gastrointestinal tract scarring following iron-induced corrosivedamage.

References:

Liebelt EL, Kronfol R. Acute iron poisoning. UpToDate Online 16.3. 2008. Available at:http://www.utdol.com/online/content/topic.do?topicKey=ped_tox/4912&selectedTitle=1~150&sou

Copyright © 2010 by the American Academy of Pediatrics page 5

Dr_Faqehi

Page 75: 04 Poisoning

2010 PREP SA on CD-ROM

rce=search_result11

Manoguerra AS, Erdman AR, Booze LL, et al. Iron ingestion: an evidence-based consensusguideline for out-of-hospital management. Clin Toxicol (Phila). 2005;43:553-570. Abstractavailable at: http://www.ncbi.nlm.nih.gov/pubmed/16255338

Spanierman C. Toxicity, iron. eMedicine Specialties, Emergency Medicine, Toxicology. 2007.Available at: http://www.emedicine.com/emerg/topic285.htm

Copyright © 2010 by the American Academy of Pediatrics page 6

Dr_Faqehi

Page 76: 04 Poisoning

2010 PREP SA on CD-ROM

Question: 64

A father brings his 2-year-old son to the emergency department in status epilepticus. He reportsthat the boy spent several hours in the garage with him while he was repairing the car. Onquestioning, the father states that over the course of the afternoon the child seemed sleepierthan usual, then became lethargic, vomited, and seemed like he was "drunk." On the way to thehospital he began having seizures. In the emergency department, the boy is given a dose oflorazepam to stop the seizure and is endotracheally intubated because of respiratory

depression. His initial laboratory results are:

•Sodium, 138 mEq/L (138 mmol/L)

•Potassium, 4.9 mEq/L (4.9 mmol/L)

•Chloride, 100 mEq/L (100 mmol/L)

•Bicarbonate, 6 mEq/L (6 mmol/L)

•Glucose, 120 mg/dL (6.7 mmol/L)

•Blood urea nitrogen, 10 mg/dL (3.6 mmol/L)

•Calcium, 5.5 mEq/L (5.5 mmol/L)•Serum osmolality, 335 mOsm/kg (335 mmol/kg)

Of the following, the MOST likely cause of this child’s clinical condition is ingestion of

A. ethylene glycol

B. gasoline

C. motor oil

D. organophosphate insecticide

E. turpentine

Copyright © 2010 by the American Academy of Pediatrics page 7

Dr_Faqehi

Page 77: 04 Poisoning

2010 PREP SA on CD-ROM

Preferred Response: ACritique: 64

The progressive lethargy, ataxia, seizures, anion gap metabolic acidosis of 30 mEq/L (ItemC64A), and osmolar gap of 53 mmol/L (Item C64B) described for the boy in the vignette arehighly suggestive of alcohol poisoning. This is a particular diagnostic possibility because the boymay have had access in the garage to such potential toxic alcohols as ethylene glycol(antifreeze) and methanol (windshield wiper fluid). The hypocalcemia suggests ethylene glycolexposure because the metabolism of ethylene glycol uses the patient’s calcium stores to createcalcium oxalate, which is excreted in the urine as crystals (Item C64C). Other findings inethylene glycol poisoning may include flank pain, hematuria, and acute renal failure.

Rapid diagnosis is critical for a patient who has symptomatic ethylene glycol poisoningbecause delay in treatment can lead to renal damage, cerebral herniation, multiple organ systemfailure, and death. Often, initial treatment is based on clinical suspicion before alcohol values areavailable. Indirect laboratory evidence of alcohol toxicity includes an anion gap acidosis and anosmolar gap greater than 10 mmol/L. Initial treatment involves stabilization of vital functions,administration of sodium bicarbonate to correct acidosis, and administration of the antidotefomepizole (or ethanol, if fomepizole is unavailable). Fomepizole inhibits alcohol dehydrogenase,which metabolizes the nontoxic parent alcohols into their toxic byproducts. Because alcoholsare absorbed so quickly from the gastric mucosa, there is little role for gastrointestinaldecontamination. Hemodialysis is indicated for severe poisonings.

Many household products are toxic and frequently accessible to young children. Gasolineand turpentine are volatile hydrocarbons that cause pulmonary injury after aspiration. Motor oilalso is a hydrocarbon, but because of its high viscosity and low volatility, it poses little risk foraspiration or toxicity. Organophosphate insecticides inhibit acetylcholinesterase and cause acholinergic crisis manifested by bradycardia, hypersalivation, bronchorrhea, diarrhea, andmuscle weakness.

References:

Keyes DC. Toxicity, ethylene glycol. eMedicine Specialties, Emergency Medicine, Toxicology.2007. Available at http://www.emedicine.com/emerg/topic177.htm

Sharman M, Sarnaik AP. Approach to the child with metabolic acidosis. UpToDate Online 16.3.2008. Available at:http://www.utdol.com/online/content/topic.do?topicKey=pedineph/16228&selectedTitle=2~150&source=search_result

Sivilotti MLA, Winchester JF. Methanol and ethylene glycol poisoning. UpToDate Online 16.3.2008. Available at:http://www.utdol.com/online/content/topic.do?topicKey=ad_tox/8204&selectedTitle=2~105&source=search_result

Copyright © 2010 by the American Academy of Pediatrics page 8

Dr_Faqehi

Page 78: 04 Poisoning

2010 PREP SA on CD-ROM

Critique: 64

Copyright © 2010 by the American Academy of Pediatrics page 9

Dr_Faqehi

Page 79: 04 Poisoning

2010 PREP SA on CD-ROM

Critique: 64

Copyright © 2010 by the American Academy of Pediatrics page 10

Dr_Faqehi

Page 80: 04 Poisoning

2010 PREP SA on CD-ROM

Critique: 64

Calcium oxalate crystals (arrows). Reprinted with permission from Schumann GB, Friedman SK.Wet Urinalysis. Chicago, Ill: American Society for Clinical Pathology; 2003.

Copyright © 2010 by the American Academy of Pediatrics page 11

Dr_Faqehi